Fluidos en Reposo

APUNTES DE FISICA MsC. JESUS ROBERTO GAVIDIA IBERICO 1 CAPITULO 12 FLUIDOS EN REPOSO Los tres estados comunes o fase

Views 298 Downloads 11 File size 1MB

Report DMCA / Copyright

DOWNLOAD FILE

Recommend stories

Citation preview

APUNTES DE FISICA

MsC.

JESUS ROBERTO GAVIDIA IBERICO

1

CAPITULO 12 FLUIDOS EN REPOSO Los tres estados comunes o fases de la materia son sólidos, liquido y gaseoso. Por lo regular distinguimos estas tres fases de la siguiente manera: Un sólido conserva una forma y un tamaño definido; incluso si se aplica una fuerza grande a un sólido, éste no cambia su forma de inmediato ni su volumen. Un liquido no puede sufrir un esfuerzo cortante y no puede conservar un forma definida (toma la forma del recipiente que lo contiene) pero, como un sólido, no es inmediatamente compresible y su volumen puede cambiar de manera significativa sólo mediante una fuerza muy grande. Un gas no tiene forma ni volumen definidos (se expandirá hasta llenar el recipiente que lo contiene). Por ejemplo, cuando se bombea aire en el neumático de un automóvil, éste no se va toda a la base del neumático como lo haría un líquido; por el contrario, llena todo el volumen del neumático. Como los líquidos y los gases no conservan una forma definida, ambos tienen la capacidad de fluir; por esto a menudo se denominan colectivamente como fluidos. La división de la materia en tres estados no siempre es simple. Por ejemplo, ¿Cómo debería clasificarse la mantequilla? Más aún, puede distinguirse un cuarto estado de la materia, el plasma, que ocurre sólo a temperaturas muy elevadas, donde hay electrones libres de sus átomos que entonces se convierten en iones. Algunos científicos creen que los llamados coloides (suspensiones de pequeñas partículas en un líquido) también debieran considerarse un estado distinto de la materia. Sin embargo, para nuestros propósitos estaremos interesados principalmente en los tres estados ordinarios de la materia. En capítulos anteriores analizamos el movimiento de los cuerpos sólidos. En este capitulo analizaremos el comportamiento de los fluidos en reposo y en el capitulo siguiente, la de os fluidos en movimiento. 12.1.

DENSIDAD ESPECIFICA Y RELATIVA

En algunas ocasiones se dice que el hierro el “más pesado” que la madera. Esto no puede ser cierto en realidad puesto que un gran tronco obviamente pesa más que un clavo de hierro. Lo que podemos decir es que el hierro es más denso que la madera. La densidad, ρ, de una sustancia se define como su masa por unidad de volumen:



m dm  V dV

(12.1)

donde m es la masa de una cantidad de sustancia cuyo volumen es V. La densidad es una propiedad característica de una sustancia; los objetos hechos de una sustancia dada, por ejemplo hierro, pueden tener cualquier tamaño o masa, pero la densidad será la misma para todos. La unidad S.I. de la densidad es kg/m3. En ocasiones las densidades se dan en g/cm3. Nótese que como 1 kg/m3 = 1000 g/(100 cm)3 = 10-3 g/cm3, una densidad dada en g/cm3 debe multiplicarse por 1000 para dar el resultado en kg/m3. de manera que la densidad del aluminio es ρ = 2,70 g/cm3 que es equivalente a 2 700 kg/m3. Las densidades de una variedad de sustancias se proporcionan en la tabla 12.1. La temperatura y la presión afectan la densidad de las sustancias. (Aunque el efecto es pequeño en los sólidos y en los líquidos) y por tanto se especifica la temperatura en la tabla 12.1.

1 g / cm3  1kg / m3 Tabla 12.1. Densidades de sustancias Sustancia: ρ Sustancia Sólidos (kg/m3) líquidos Aluminio 2 700 Agua (20º C) Hierro y acero 7 800 Plasma sanguíneo Cobre 8 900 Sangre Plomo 11 300 Agua de mar Oro 19 300 Mercurio Concreto 2 300 Alcohol etílico

ρ (kg/m3) 1 000 1 030 1 050 1 025 13 600 790

Sustancia gases Aire Helio Bióxido de carbono Agua (Vapor) (100º C)

ρ (kg/m3) 1,29 0,179 1,98 0,598

APUNTES DE FISICA

2

Granito Madera (común) Hielo Hueso

2 700 300 – 900 917 1 700 – 2 000

MsC.

JESUS ROBERTO GAVIDIA IBERICO

Gasolina

680

Ejemplo 12.1. ¿Cuál es la masa de una esfera de plomo de 0,500 m de radio? Solución El volumen de la esfera es:

4 4 V   r 3   0,500m3  0,523 m3 3 3 de la Tabla 12.1., la densidad del plomo es: ρ = 11 300 kg/m3, de modo que tenemos de la ecuación 12.1:







m   V  11300kg / m3 0,523m3  5910kg la densidad relativa o gravedad especifica de una sustancia se define como la razón de la densidad de esa sustancia entre la densidad del agua a 4º C. La densidad relativa (ρr) es un número sin dimensiones ni unidades. Como la densidad del agua es 1,000 g/cm3 = 1000 kg/m3, la densidad relativa de cualquier sustancia será precisamente igual, desde el punto de vista numérico, a su densidad especificada en g/cm3, o 10-3 veces su densidad especificada en kg/m3. Por ejemplo (véase tabla 12.1) la densidad relativa del plomo es 11,3 y la del alcohol 0,79.

r 

c H O

c   r  H O



2

2

12.2.

PRESION DE UN FLUIDO

La presión se define como la fuerza por unidad de área, donde la fuerza F se considera que actúa en forma perpendicular a la superficie de área A:

P

F dF  A dA

(12.2)

La unidad de presión en el SI es N/m2, esta unidad recibe el nombre oficial de Pascal (Pa): 1 Pa = 1 N/m2. No obstante, por sencillez, nosotros usaremos con más frecuencia N/m2. Otras unidades que se usan en ocasiones son dinas/cm2, lb/pie2 (algunas veces abreviada “psi”), y kg/cm2 (como si los kilogramos fueran una fuerza: esto es 1 kg/cm2 = 9,8 N/cm2 = 9,8 x 104 N/m2). Las últimas dos se usan con frecuencia en los manómetros de las llantas. Más adelante encontraremos otras unidades. Como un ejemplo de cómo se calcula la presión, una persona de 60 kg cuyos dos pies cubre un área de 500 cm2 ejerce una presión de:

P





F m g 60 kg  9,8 m / s 2 N    12 x103 2 2 A A 0,050 m m

sobre el suelo. Si la persona se para en un pie, la fuerza es la misma pero el área será la mitad, de tal manera que la presión será el doble que la anterior: 24 x 103 N/m2. En concepto de presión es particularmente útil cuando se tratan fluidos, razón por la cual lo introducimos en este capitulo. Es un hecho experimental que un fluido ejerce una presión en todas direcciones. Esto lo saben muy bien los nadadores y los clavadistas quienes sienten la presión del agua en todas las partes del cuerpo. En un punto en particular de un fluido en reposo, la presión es la misma en todas direcciones. Esto se ilustrar en la figura 12.1; considérese un pequeño cubo de fluido que es tan reducido (infinitesimal) que podemos ignorar la fuerza de gravedad que actúa sobre él. Entonces la presión sobre un lado de él debe ser igual a la presión en el lado opuesto. Si esto no ocurriera, la fuerza neta sobre este cubo no seria cero y éste se moverá hasta que la presión se igualara; si el fluido no está fluyendo, entonces las presiones deben ser iguales.

APUNTES DE FISICA

MsC.

JESUS ROBERTO GAVIDIA IBERICO

3

Figura 12.1. La presión es la misma en todas las direcciones en un fluido a una profundidad dada; si no fuera así. El fluido estaría en movimiento

Otra propiedad importante de un fluido en reposo es que la fuerza debida a presión de un fluido siempre actúa de manera perpendicular a cualquier superficie con la que esté en contacto. Si hubiera una componente de la fuerza paralela a la superficie, como se muestra en la figura 12.2, entonces de acuerdo con la tercera ley de newton, la superficie ejercería una fuerza de atracción sobre el fluido que también tendría una componente paralela a la superficie; esta componente provocaría que el fluido fluyera, en contradicción con nuestra suposición de que el fluido se encuentra en reposo.

Figura 12.2. La fuerza que ejerce un fluido que no se mueve sobre una superficie sólida es perpendicular a ésta; esto es’ F  = 0

A continuación calcularemos de manera cuantitativa cómo varía la presión con la profundidad en un liquido de densidad uniforme. Consideres un punto que está a una profundidad h bajo la superficie del liquido (esto es, la superficie está a una altura h encima de ese punto) como se muestra en la figura 12.3.

Figura 12.3. Cálculo de la presión en un líquido a una profundidad h

La presión debida al líquido en esta profundidad h se debe al peso de la columna de liquido encima de ella. Así, la fuerza que actúa sobre el área es:

F  m g   Ah g

donde A h es el volumen de la columna, ρ la densidad del liquido (supuesta constante) y g la aceleración de la gravedad. La presión, P, es entonces:

P

F  Ah g    gh A A

P   gh

(Liquido)

(12.3)

4

APUNTES DE FISICA

MsC.

JESUS ROBERTO GAVIDIA IBERICO

Así, la presión es directamente proporcional a la densidad del líquido y a la profundidad dentro del líquido. En general, la presión a profundidades iguales dentro de un líquido uniforme es la misma. La ecuación 12.3 nos dice cuál es la presión a una profundidad h en el líquido, debida al mismo liquido. Pero ¿qué ocurre cuando hay una presión adicional ejercida en la superficie del líquido, como la presión de la atmósfera? Los gases son bastantes compresibles y por tanto su densidad puede variar en forma significativa con la profundidad; los líquidos también pueden comprimirse, aunque con frecuencia podemos ignorar la variación de su densidad. (Una excepción se encuentra en las profundidades del océano, donde el enorme peso del agua de encima comprime de manera significativa el agua e incrementa su densidad). Para cubrir estos y otros casos, a continuación trataremos el caso general de determinar como varia la presión en el seno de un fluido con la profundidad. Variación de la presión en un fluido con la profundidad Consideres cualquier fluido y determinemos la presión a cualquier altura y por encima de algún punto de referencia, como se muestra en la figura 12.4.

Figura 12.4. Fuerzas sobre una rebanada aplanada de fluido para determinar la presión P a una altura y en un fluido

Dentro del fluido, a una altura y, consideramos una pequeña y aplanada rebanada de volumen del fluido cuya área es A y cuyo grosor (infinitesimal) es dy, como se muestra. Sea P la presión que actúa hacia arriba sobre su superficie inferior (en la altura y); la presión que actúa hacia abajo en la superficie superior de la pequeña rebanada (en la altura y + dy) es entonces P + dP. La presión del fluido que actúa sobre nuestra rebanada ejerce una fuerza igual a PA hacia arriba sobre la rebanada y (P + dP) hacia abajo sobre ésta; la única otra fuerza que actúa verticalmente sobre la rebanada es la fuerza (infinitesimal) de la gravedad dW que está dada por:

dW  d m g   g dV   g Ady donde ρ es la densidad del fluido en la altura y. Como se supone que el fluido se encuentra en reposo, nuestra rebanada está en equilibrio de modo que la fuerza neta sobre ésa debe ser cero. En consecuencia:

PA  P  dP A   g Ady  0 que cuando se simplifica se convierte en:

dP   g dy

(12.4)

Esta relación nos dice cómo varia la presión con la altura dentro del fluido. El signo negativo indica que la presión disminuye con un incremento en la altura; o que la presión aumenta con la profundidad (cuando disminuye la altura).

APUNTES DE FISICA

MsC.

JESUS ROBERTO GAVIDIA IBERICO

5

Figura 12.5. La presión en un liquido de densidad ρ a una profundidad h = (y2 – y1) es P = P0 + ρgh, donde P0 es la presión externa en la parte superior de la superficie del liquido

Si la presión en el fluido es P1 en la altura y1 y a la altura y2 es P2, entonces podemos integrar la ecuación 12.4 para obtener: P2

y2

P1

y1

 dP     g dy y2

P2  P1     g dy

(12.5)

y1

esta es una relación general y la aplicamos ahora a dos casos especiales: (a) la presión en los líquidos de densidad constante y (2) las variaciones de presión en la atmósfera terrestre Presión en los líquidos de densidad constante Para los líquidos en que se puede ignorar cualquier variación de la densidad, ρ = constante y la ecuación 12.5 se integra de manera inmediata: (12.6a) P2  P1   g y2  y1





Para situaciones ordinarias de un líquido en un recipiente abierto (como el agua de una alberca, un lago o el océano existe una superficie libre en la parte superior. Y es conveniente medir las distancias desde esta superficie superior; esto es, hacemos que h sea la profundidad en el liquido donde h = y2 – y1 como se muestra en la figura 12.5. Si hacemos que y2 sea la posición de la superficie superior, entonces P2 representa la presión atmosférica, P0, en dicha superficie. Entonces la presión P (= P1) en una profundidad h en el fluido es, de acuerdo a la ecuación 12.6a: P  P0   g h (12.6b) Nótese que la ecuación 12.6b es igual que la 12.3 para la presión del liquido, más la presión P0 debida a la atmósfera de encima. Ejemplo 12.2. La superficie del agua en un tanque de almacenamiento se encuentra a 30 m sobre la llave del agua de la cocina de una casa. Calcule la presión del agua en la llave. Solución La presión atmosférica actúa tanto en la superficie del agua en el tinaco como en el agua que sale de la llave. La diferencia de presión adentro y afuera de la llave será:







P   g h  1000kg / m3 9,8 m / s 2 30 m  2,9 x105 N / m2

APUNTES DE FISICA

6

MsC.

JESUS ROBERTO GAVIDIA IBERICO

La altura h se conoce también como carga de presión o altura de presión. En este ejemplo, la altura de presión del agua es de 30 m.

EJEMPLO 13.3. Fuerza en la ventana de un acuario. Calcule la fuerza debida a la presión del agua que se ejerce sobre la ventana de observación de un acuario de 1,0 m x 3,0 m, cuyo borde superior está a 1,0 m debajo de la superficie del agua (ver figura) PLANTEAMIENTO. A una profundidad h, la presión debida al agua está dada por la ecuación 13.6.b. Divida la ventana en franjas horizontales de longitud ℓ = 3,0 m y grosor dy, como se ilustra en la figura. Elegimos un sistema de coordenadas con y = 0 en la superficie del agua, donde y es positivo hacia abajo. (De esta manera, el signo menos en la ecuación 13.6.a se vuelve positivo, o bien, usamos la ecuación 13.6.b con y = h. la fuerza debida a la presión del agua sobre cada franja es dF = P dA = gy dy.

SOLUCIÓN. La fuerza total sobre la ventana está dada por la integral: y 22, 0 m

1

  g  y dy  2  g  ( y

y11,0 m

2 2





1  y12 )  (1000 kg / m3 ) (9,80 m / s 2 ) (3,0 m) (2,0 m) 2  (1,0 m) 2  44 000 N 2

Para verificar la respuesta, es conveniente hacer una estimación: multiplique el área de la ventana (3,0 m2) con la presión en la mitad de la ventana (h = 1,5 m) utilizando la ecuación 13.3, P = gh = (1000 kg/m3)(9.80 m/s2)(1,5 m)  1,5 x 104 N/m2. De manera que F = P A  (1,5 x 104 N/m2) 83,0 m) (1,0 m)  4,5 x 104 N. ¡Bien!

FUERZAS SOBRE PAREDES O COMPUERTAS Las fuerzas horizontales causadas por la presión sobre superficies que encierran fluido, aumentan linealmente con la profundidad, de modo que se tienen fuerzas distribuidas no uniformes actuando sobre ellas. Como se explicó en el capítulo de Estática, la resultante de ese sistema de fuerzas paralelas es en general una fuerza paralela aplicada en un punto arbitrario, más el torque de todas esas fuerzas distribuidas respecto a ese mismo punto. Es sin embargo conveniente calcular la resultante de esas fuerzas en un cierto punto, llamado centro de presión, respecto al cual el torque de las fuerzas distribuidas es nulo. Explicaremos entonces la forma de hacerlo. Esto requiere sin embargo de elementos de cálculo integral que trataremos de omitir. Para el caso de compuertas y situaciones similares, la fuerza debido a la presión atmosférica actúa por ambos lados, y entonces la omitiremos del análisis por no contribuir en forma neta a la fuerza horizontal actuando sobre la superficie. La figura siguiente ilustra una situación típica, donde por el interior de una superficie hay un fluido y por el exterior está la atmósfera.

APUNTES DE FISICA

MsC.

JESUS ROBERTO GAVIDIA IBERICO

7

El término de la profundidad y la fuerza neta que actúa a esa profundidad sobre el elemento de área w y altura dy es: d F  p w dy   g w y dy Entonces se tiene una fuerza distribuida cuya magnitud por unidad de longitud varía linealmente de la forma:

dF   wg y dy

(

N ) m

El cálculo de la fuerza resultante dependerá de la forma de la superficie que se considere SUPERFICIE RECTANGULAR El caso más importante es si la superficie es rectangular como se indica en la figura que sigue donde se desea evaluar la fuerza resultante de las fuerzas distribuidas entre y1 e y2. Como vimos en el capítulo de fuerzas, la resultante y el punto de aplicación corresponden al área y al centroide de la figura correspondiente a la fuerza distribuida entre y1 e y2.

8

APUNTES DE FISICA

MsC.

JESUS ROBERTO GAVIDIA IBERICO

Como sabemos el área es:

1 (a  b) ( y2  y1 ) 2 y el centroide está (medido desde y1 hacia abajo):

1 (a  2 b) ( y2  y1 ) 3 a b donde a y b son las fuerzas por unidad de longitud en y1 e y2:

a   w g y1 b   w g y2

así entonces la fuerza resultante es:

APUNTES DE FISICA

F

MsC.

1  w g y1   w g y2 2

JESUS ROBERTO GAVIDIA IBERICO

9

 y2  y1   1  w g  y2  y1  2

y su punto de aplicación será:

y P  y1 

1 (a  2 b) 2 y12  y2 y1  y22 ( y2  y1 )  3 ab 3 y1  y2

En particular si la superficie está entre y1 = 0 e y2 = h resulta:

yP 

2 h 3

SUPERFICE DE FORMA ARBITRARIA Si la superficie no es rectangular, como se ilustra en la figura que sigue es necesario recurrir al cálculo integral. La fuerza actuando sobre el elemento de área dA = dx dy indicado en la figura será:

d F   g y dA   g y d x d y

De modo que la fuerza resultante será:

F   g  y dx dy A

y la posición de centro de fuerza estará dada por sus coordenadas:

xP 

 x dF A

F

10

APUNTES DE FISICA

MsC.

yP 

JESUS ROBERTO GAVIDIA IBERICO

 y dF A

F

que pueden escribirse:

xP

 x y dx dy   y dx dy A

A

 y dx dy   y dx dy 2

yP

A

A

integrales que podrían hacerse si se conoce la forma del área. Como usted no domina aún el tema de las integrales, nos e preocupe demasiado por esta sección. FUERZA SOBRE UNA SUPERFCIE DE FORMA RECTANGULAR INCLINADA En una sección anterior se calculó la fuerza resultante y centro de la fuerza para un área vertical de sección rectangular. Para una sección rectangular inclinada un ángulo  con la vertical, el cálculo es muy parecido, pero ahora, el eje OY está inclinado, luego debemos tomar:

a   w g y1 cos  b   w g y2 cos  y luego resultarán:

F





1  w g y22  y12 cos  2

y su punto de aplicación será:

yP 

2 y12  y2 y1  y22 3 y1  y2

Note que la expresión para el centro de fuerza es la misma:

APUNTES DE FISICA

MsC.

JESUS ROBERTO GAVIDIA IBERICO

11

EJEMPLO. La fuerza sobre un dique La altura del agua detrás de un dique de ancho w es H (Fig. 15.7). Determine la fuerza resultante ejercida por el agua sobre el dique.

Figura. Como la presión varía con la profundidad, la fuerza total ejercida sobre una presa debe ser obtenida de la expresión F = p dA, donde dA es el área de la banda oscura. Solución Puesto que la presión varía con la profundidad, no se puede calcular la fuerza simplemente multiplicando el área por la presión. El problema puede resolverse encontrando la fuerza dF ejercida sobre una estrecha tira horizontal a una profundidad h, e integrando después la expresión para encontrar la fuerza total. Imagine un eje y vertical, con y = 0 en el fondo del dique y la tira a una distancia y sobre el fondo. Se puede usar la ecuación 15.4 para calcular la presión a la profundidad h; se omite la presión atmosférica porque actúa en los dos lados del dique: P =  g h =  g (H  y) Mediante la ecuación 15.2 se encuentra que la fuerza ejercida sobre la tira sombreada de área dA = w dy es: dF = P dA =  g (H  y) w dy Por tanto, la fuerza total sobre el dique es: H 1 F   dF    g ( H  y)w dy   g w H 2 2 0

Advierta que el espesor del dique aumenta con la profundidad, como se muestra en la figura 15.7. Este diseño considera las presiones cada vez mayores que el agua ejerce sobre el dique a mayor profundidad. Ejercicio: Encuentre una expresión para la presión promedio sobre el dique a partir de la fuerza total ejercida por el agua sobre el dique. Respuesta:

1 gH. 2

EJEMPLOS 1.

La compuerta de la figura tiene 2 m de ancho y contienen agua. Si el eje que soporta la compuerta que pasa por A soporta un par máximo de 150 kN . m, determine la máxima altura h que puede tener el agua.

12

APUNTES DE FISICA

MsC.

JESUS ROBERTO GAVIDIA IBERICO

SOLUCIÓN El perfil de presión que actúa sobre la compuerta se ilustra en la figura que sigue:

Usaremos fórmulas:

yP 

FP 

2 2 2 y1  y1 y2  y2 3 y1  y2





1  w g y2 2  y12 cos(90º  ) 2

siendo

y2 

h sen 

y1 

h sen 

De manera que la fuerza resultante es: 2  h  2  h   1 1    FP   w g   L     wg L 2 h  L sen   2  sen    sen    2

y su punto de aplicación resultará: 2 2 2 y1  y1 y2  y2 2 3 h 2  3 h L sen   L2 sen2  yP   3 y1  y2 3 2 h sen   L sen2 

El torque será de magnitud.

APUNTES DE FISICA

MsC.

1 2



JESUS ROBERTO GAVIDIA IBERICO

13

h 2 3 h 2  3 h L sen   L2 sen2     2 sen  3 2 h sen   L sen   

 A  FP  y2  y P     w g L 2 h  L sen   

A 

1 3 h  2 L sen   w g L2 6

Numéricamente w = 2 m,  = 1000 kg/m3, g = 9,80 m/s2 , L =

(2,1 m) 2  (2,8 m) 2 = 3,5 m. L sen  = 2,8 m,

calculamos:

A 

1 3h  2 L sen   w g L2  1,25 x105 h  2,286667 x105  150 x103 6

de donde se obtiene: h = 3,091 m 2.

Determínese el par que se requiere hacer en A para sostener la compuerta indicada cuyo ancho, perpendicular al papel es w = 2 m.

SOLUCIÓN Si z indica la posición de la compuerta medida desde A hacia abajo, entonces numéricamente ( = 1000 kg/m3, g = 9,80 m/s2, w = 2 m)

p  10 000 (4  z) N/m2 y la fuerza por unidad de longitud será:

F  20 000 (4  z) N/m L Su resultante y punto de aplicación será calculada igual que en el problema anterior con:

1 F  (2) (20 000 x 4  20 000 x 6)  200 000 N 2 y su punto de aplicación es:

APUNTES DE FISICA

14

zC 

MsC.

JESUS ROBERTO GAVIDIA IBERICO

2 (20 000 x 4)  2 (20 000 x 6)  1,067 m 3 20 000 x 4  (20 000 x 6)

de modo que el torque es:

 A  200 000 x1,067  2,134 x105 N . m Note de nuevo que integrando es mucho más directo: 2

 20 000 (4  z) z dz  2,13 x105 N . m 0

3.

Determine la ubicación “y” del pivote fijo A de manera que justo se abra cuando el agua está como se indica en la figura.

SOLUCIÓN Si h indica una coordenada de posición medida desde la superficie del agua hacia abajo, entonces la presión en un punto ubicado a esa profundidad es:

P   gh (la presión atmosférica actúa por ambos lados ys e cancela). Para que la compuerta justo se abra, el punto de aplicación medida desde el punto más alto de la compuerta puede escribirse:

zC 

1 ( w g h1 )  2 ( w g h2 ) 1 h1  2 h2 L  L 3  w g h1  w g h2 3 h1  h2

entonces

2  2 (2) 1 (1)  0,56 m 3 1 2 por lo tanto: y = (1  0,56) m = 0,44 m

APUNTES DE FISICA

MsC.

JESUS ROBERTO GAVIDIA IBERICO

15

Variación de presión en la atmósfera terrestre A continuación aplicaremos la ecuación 12.4 o 12.5 a los gases. La densidad de los gases normalmente es muy pequeña, de tal manera que la diferencia en la presión en diferentes alturas puede ignorarse; si y2 – y1 no es muy grande (razón por la que en el ejemplo 12.2 pudimos ignorar la diferencia en la presión del aire entre la llave y la parte superior del tinaco). Sin duda, para la mayor parte de los recipientes comunes de gas, podemos suponer que la presión es la misma en todo el recipiente. Sin embargo, si y2 - y1 es muy grande, no podemos hacer esta suposición. Un ejemplo interesante lo constituye la atmósfera terrestre cuya presión en el nivel del mar es de unos 1,013 x 105 N/m2 y disminuye con la altura. Ejemplo 12.3. (a) Determine la variación en la presión de la atmósfera de la Tierra en función de la altura y sobre el nivel del mar, suponiendo que g es constante y que la densidad del aire es proporcional a la presión. (Esta última suposición no es en extremo precisa, en parte debido a que los efectos de la temperatura son importantes. (b) ¿A que altura la presión del aire es igual a la mitad de la presión al nivel del mar? Solución (a) Como ρ es proporcional a P, tenemos:

 P  0 P0 donde P0 = 1,013 x 105 N/m2 es la presión atmosférica al nivel del mar y ρ0 = 1,29 kg/m3 es la densidad del aire al nivel del mar a 0o C (Tabla 12.1). de la ecuación 12.4 tenemos:

dP   g dy

 P 0  dP  g   dy  P0  de modo que:

 dP   0 g dy P P0

Integramos ésta desde y = 0, P = P0 hasta la altura y donde la presión es P:

0 g y dP    P0 0 dy P0 P P

ln

g P  0 y P0 P0

o bien:

P  P0 e

 g  0  y  P0 

Así, con base en nuestras suposiciones, encontramos que la presión del aire en nuestra atmósfera disminuye de manera aproximadamente exponencial con la altura. (Nótese que la atmósfera no tiene un limite superior definido, de manera que no existe ningún punto natural desde el que pueda medirse la profundidad de la atmósfera, como en el caso de un liquido). (b) la constante (ρ0g/P0) tiene el valor:

0 g P0

1,29 kg / m 9,8 m / s   1,25 x 10  3

2

5

1,013 x 10 N / m

2

4

1 m

16

APUNTES DE FISICA

MsC.

JESUS ROBERTO GAVIDIA IBERICO

Entonces, cuando hacemos P = ½ P0 tenemos:  4 1 1 P0  P0 e (1,24x10 m ) y 2

o bien:

y

ln(1/ 2)  0,693   5 550 m 4  1,24 x10  1,24 x104 m1

De este modo, la presión atmosférica disminuye hasta la mitad de su valor al nivel del mar a una altura de unos 5 550 m (aproximadamente 18 000 pies); no es sorprendente que los montañistas utilicen a menudo tanques de oxigeno a grandes alturas. 12.3.

PRESION ATMOSFÉRICA Y PRESION MANOMÉTRICA

La presión de la atmósfera terrestre varía con la altura, como se ha visto. Pero aun en un lugar determinado, varia ligeramente de acuerdo con el clima. Como ya habíamos mencionado antes, al nivel del mar la presión promedio de la atmósfera es de 1,013 x 105 N/m2 (o 14,7 lb/pie2). Este valor se usa para definir una unidad de presión usada en forma corriente, la atmósfera (que se abrevia atm): 1 atm = 1,013 x 105 N/m2 = 1,013 x 105 Pa Otra unidad de presión utilizada en ocasiones (en meteorología y en mapas de clima) es el bar que se define: 1 bar = 1,00 x 105 N/m2; así, la presión atmosférica estándar es ligeramente mayor que un bar. La presión debida al peso de la atmósfera se ejerce sobre todos los objetos que están inmersos en este gran océano de aire y por supuesto sobre nuestro cuerpo. ¿Cómo puede el cuerpo humano soportar la enorme presión sobre su superficie? La respuesta se encuentra en el hecho de que las células vivas mantienen una presión interna que equilibra de manera exacta la presión externa. De modo similar, la presión interior de un balón equilibra la presión exterior de la atmósfera. Una llanta de un automóvil, debido a su rigidez, puede mantener presiones mucho mayores que la presión externa. No obstante, debe tenerse cuidado al determinar la presión de una llanta o de cualquier otro recipiente de gas debido a que los manómetros, y la mayor parte de otros medidores de presión, registran la presión sobre y encima de la presión atmosférica. Esta se conoce como la presión manométrica (Pm). Por eso, para obtener la presión absoluta P, debe sumarse la presión atmosférica, P0, a la manométrica Pm:

P  Pm  P0 Como ejemplo mencionaremos que si un manómetro de neumáticos registra 220 kPa, la presión real dentro del mismo es 220 kPa + 100,3 kPa = 320,3 kPa. Esto es equivalente a una 3,2 atm (2,3 atm la presión atmosférica). 12.4.

MEDICION DE LA PRESION

Se han inventado numerosos aparatos para medir la presión, algunos de ellos se muestran en la figura 12.6. El más simple de todos es el manómetro abierto (Figura 12.6a) que es un tubo en forma de U parcialmente lleno con un liquido manométrico, por lo regular mercurio o agua. La presión P que se mide se relaciona con la diferencia en altura de dos niveles del líquido mediante la relación:

P  P0   g h donde P0 es la presión atmosférica y ρ es la densidad del liquido manométrico. Nótese que la cantidad ρgh es la “presión manométrica”, [la cantidad en que P sobrepasa la presión atmosférica (Sección 12.3)] Si el liquido de la columna izquierda estuviera más abajo que el de la derecha, esto indicaría que P seria menor que la presión atmosférica (y h seria negativa)

APUNTES DE FISICA

MsC.

JESUS ROBERTO GAVIDIA IBERICO

17

Figura 12.6. Medidores de presión: (a) manómetro de tubo abierto; (b) manómetro de Bourdon; (c) manómetro aneroide

En vez de calcular el producto ρgh, es común especificar nada más la altura h. de hecho, las presiones se especifican en ocasiones como “milímetros de mercurio” (mm Hg) y otras veces como “milímetros de agua” (mm H2O). La unidad mm Hg es equivalente a la presión de 133 N/m2, puesto que 1,00 mm = 1,00 x 10-3 m y :

gh  1000kg / m3 9,8 m / s 2 1.00 x103 m  1,33 x102 N / m2 La unidad mmHg se llama también “torr” en honor a Evangelista Torricelli (1608 – 1657) quien invento el barómetro (véase más adelante). En la Tabla 12.2 se proporcionan los factores de conversión entre las diferentes unidades de presión. Es importante que sólo se usen N/m2 = Pa, la unidad propia del SI, en los cálculos en que intervienen otras cantidades especificadas en unidades del SI. Tabla 12.2. Factores de conversión entre diferentes unidades de presión En términos de 1 Pa = 1 N/m2 1 atm = 1,013 x 105 N/m2 1atm = 1,013 x 105 Pa = 101,3 kPa

1 bar = 1,000 x 105 /m2 1 dina/cm2 = 0,1 N/m2 1 kg/cm2 = 9,85 x 104 N/m2 1 lb/pie2 = 6,90 x 103 N/m2 1 lb/pie2 = 47,9 N/m2 1 cm Hg = 1,33 x 103 N/m2 1 mm Hg = 133 N/m2 1 torr = 133 N/m2 1 mm H2O (4o C) = 9,81 N/m2

Relativos a 1 atm 1 atm = 1,013 x 105 N/m2 1 atm = 1,103 bar 1 atm = 1,013 x 106 dinas/cm2 1 atm = 1,03 kg/cm2 1 atm = 14,7 lb/pulg2 1 atm = 2,12 x 103 lb/pie2 1 atm = 76 cm Hg 1 atm = 760 mm Hg 1 atm = 760 torr 1 atm = 1,03 x 104 mm H2O (40 C)

Otros tipos de manómetros son el Bourdon (Figura 12.6b), en el que el incremento de presión tiende a enderezar el tubo delgado que está unido a una aguja y el manómetro aneroide (Figura 12.6c) en la que la aguja se conecta al extremo flexible de una delgada cámara de metal en la cual se ha hecho vació. En un tipo más complicado de manómetro, la presión a medir se aplica a un diafragma delgado de metal y la distorsión producida sobre el diafragma se detecta mediante una señal eléctrica con el uso de un llamado transductor de presión. La presión atmosférica se mide con frecuencia mediante un tipo modificado de un manómetro de mercurio cerrado. Este se llama barómetro de mercurio, Figura 12.7.

18

APUNTES DE FISICA

MsC.

JESUS ROBERTO GAVIDIA IBERICO

Figura 12.7. Barómetro de mercurio, cuando la presión del aire es 76 cmHg

El tubo de vidrio se llena con mercurio y luego se invierte dentro de un recipiente con mercurio. Si el tubo es lo suficientemente largo, el nivel de mercurio bajará, dejando un vació en la parte superior del mismo, dado que la presión atmosférica puede soportar una columna de mercurio de sólo 76 cm de altura (exactamente 76,0 cm en las condiciones normales de presión atmosférica). Esto es, una columna de mercurio de 76 cm de lato ejerce la misma presión que la atmósfera: a partir de la formula P = ρgh, con ρ = 13 600 kg/m3 para el mercurio y h = 0,760 m, tenemos:







P   g h  13600kg / m3 9,8 m / s 2 0,760m  1,013 x 105 N / m2  1,00 atm Los barómetros domésticos por lo regular son de tipo aneroide, Figura 12.6c. Un cálculo análogo al anterior mostrará que la presión atmosférica puede sostener una columna de agua de 10,3 m de alto en un tubo cuya parte superior esté vacío. Hace algunos cientos de años era una fuente de duda y de frustración el que ninguna bomba de vacío pudiera elevar el agua a una altura mayor que 10 m, sin importar lo bueno que fuera la bomba. Por ejemplo, tratar de bombear agua de los pozos profundos de las minas era enorme dificultad práctica ya que se requerían múltiples etapas para profundidades mayores que 10 m. este problema intereso a Galileo, pero fue Torricelli el primero en entender la causa. El hecho es que una bomba no succiona en realidad agua por un tubo. Es la presión del aire la que empuja el agua hacia arriba en el tubo si el otro extremo está al vacío, al igual que empuja (o mantiene) el mercurio a 76 cm de altura dentro del barómetro. 12.5.

PRINCIPIO DE PASCAL

La atmósfera de la Tierra ejerce una presión sobre todos los objetos con los que esta en contacto, entre ellos otros fluidos. La presión atmosférica que actúa sobre un fluido se transmite a través de éste. Por ejemplo, de acuerdo con la ecuación 12.6b la presión del agua a una profundidad de 100 m debajo de la superficie de un lago es:







P   g h  1000kg / m3 9,8 m / s 2 100m  9,8 x105 N / m2  9,7 atm

y la presión total en ese punto se debe a la presión del agua más la presión del aire sobre ésta; la presión total (si el lago está cerca del nivel del mar) es 9,7 atm + 1 atm = 10,7 atm. Este es sólo un ejemplo de un principio general atribuido al filósofo y científico francés Blaise Pascal (1623 – 1662). El principio de Pascal establece que la presión aplicada a un fluido confinado en un recipiente aumenta la presión del fluido en la misma cantidad. Existen numerosos aparatos que aprovechan este principio. A continuación mencionaremos dos ejemplos: los frenos hidráulicos de un auto y los gatos hidráulicos, ambos ilustrados en la Figura 12.8. En el caso de un gato hidráulico de, puede emplearse una pequeña fuerza para producir otra grande al hacer el área de un pistón (la salida) mayor que el área del otro (la entrada). Si las cantidades de entrada se representan mediante el subíndice “i” y las de salida mediante el subíndice “o” tenemos:

P0  Pi

APUNTES DE FISICA

MsC.

JESUS ROBERTO GAVIDIA IBERICO

19

F0 Fi  A0 Ai o finalmente:

F0 A0  Fi Ai F0/Fi es la “ganancia mecánica” de la prensa hidráulica y es igual a la razón de las áreas. Por ejemplo, si el área de salida de un pistón es 20 veces mayor que la del cilindro de entrada, la fuerza se multiplica por un factor de20; de este modo una fuerza de 200 lb podría levantar un auto de 4 000 lb.

Figura 12.8. Aplicaciones del principio de Pascal: (a) ascensor hidráulico; (b) frenos hidráulicos en un auto

12.6.

FLOTABILIDAD Y PRINCIPIO DE ARQUIMEDES

Los objetos sumergidos en un fluido parecen pesar menos que cuando están afuera del fluido. Por ejemplo, una piedra que se levantara con dificultad del suelo puede levantarse con facilidad del fondo de un río; cuando la roca sale a la superficie del agua de repente parece pesar mucho más. Muchos objetos, como la madera, flotan en la superficie del agua. Estos son dos ejemplos de flotabilidad . En cada uno la fuerza de la gravedad actúa hacia abajo, pero además de ésta actúa una fuerza de empuje ascendente que ejerce el líquido.

Figura 12.9. Determinación de la fuerza de empuje

La fuerza de empuje proviene del hecho de que la presión en un fluido aumenta con la profundidad, De este modo la presión ascendente sobre el fondo de la superficie de un objeto sumergido es mayor que la presión descendente sobre la superficie superior. Para observar el efecto de esto, considérese un cilindro de altura h cuya tapa y base tiene un área A y que está sumergida por completo en un fluido de densidad ρ r como se muestra en la Figura 12.9. El fluido ejerce una presión P = ρrgh1 contra la tapa del cilindro; la fuerza debida a esta presión sobre la tapa del cilindro es F1 = P1A = ρfgh1A y se dirige hacia abajo. De manera similar, el líquido ejerce una fuerza hacia arriba en el fondo del cilindro igual a F2 = P2A = ρrh2gA La fuerza neta debido a la presión del fluido, que es la fuerza de empuje FB, actúa hacia arriba y tiene una magnitud:

FB  F2  F1

20

APUNTES DE FISICA

MsC.

JESUS ROBERTO GAVIDIA IBERICO

FB   f g Ah2  h1  FB   f g Ah FB   f g V f Donde V = Ah es el volumen del cilindro. Como ρf es la densidad del fluido, el producto ρfgV = mfg es el peso del fluido que tiene un volumen igual al del cilindro. De este modo la fuerza de empuje sobre el cilindro es igual al peso del fluido que desplaza el cilindro. El resultado se cumple independientemente de la forma del objeto. Este fue descubierto por primera vez por Arquímedes (287? – 212 A.C.) y por esta rabón se conoce como el principio de Arquímedes: “la fuerza de empuje sobre un cuerpo sumergido en un fluido es igual al peso del fluido desalojado por ese objeto”. Podemos derivar el principio de Arquímedes en general mediante el siguiente argumento tan sencillo como elegante. Sobre el cuerpo de la figura irregular D mostrado en la Figura 12.10a actúa la fuerza de gravedad (su peso, W) y la fuerza de empuje FB. (si ninguna otra fuerza actúa sobre el objeto, como una mano que lo jale hacia arriba, este se moverá hacia abajo ya que W > FB) deseamos determinar FB y para esto consideramos a continuación un cuerpo del (mismo) fluido (D’ en la Figura 12-10b) de la misma forma y tamaño que el objeto original y colocado a la misma profundidad; usted podría imaginárselo como un cuerpo separado del resto del fluido por medio de una membrana transparente imaginaria. La fuerza de empuje FB sobre él será exactamente la misma que aquella sobre el objeto original dado que el fluido circundante, que ejerce F B, tiene idéntica configuración. Ahora el cuerpo de fluido D’ está en equilibrio (el fluido como un todo está en reposo); en consecuencia, FB = W’, donde W’ es el peso del cuerpo de fluido . En consecuencia, la fuerza de empuje FB es igual al peso del cuerpo del fluido cuyo volumen es igual al volumen del objeto sumergido original, que es lo que establece el principio de Arquímedes.

Figura 12-10. Principio de Arquímedes

Ejemplo 12.4. Una piedra de 70 kg se encuentra en el fondo de un lago. Su volumen es de 3 x 10 4 cm3. ¿Qué fuerza se necesita para levantarla? Solución La fuerza de empuje sobre la roca debida al agua es igual al peso de 3,0 x 10-2 m3 de agua:

FB   H2O g V







FB  1000kg / m3 9,8 m / s 2 3,0 x102 m3

FB  2,9 x 102 N El peso de la roca es:





W  m g  70 kg 9,8 m / s 2  6,9 x102 N

De ahí que la fuerza necesaria para levantarla sea:



APUNTES DE FISICA

MsC.

JESUS ROBERTO GAVIDIA IBERICO

21

W '  690 N  290 N  400 N Es como si la piedra tuviera una masa de solo:

m' 

400 N W'   41 kg g 9,8 m / s 2

Se dice que Arquímedes descubrió este principio en su baño mientras pensaba cómo podría determinar si la nueva corona del rey era de oro puro o de una aleación. El oro tiene una densidad especifica de 19,3, un poco mayor que la de la mayor parte de los metales, pero no puede determinarse de manera directa la densidad especifica o densidad dado que el volumen de un objeto de forma geométrica irregular no puede calcularse con facilidad. Sin embargo, si se pesa el objeto en el aire ( = W) y también se pesa en el agua (= W’), puede determinarse la densidad utilizando el principio de Arquímedes, como en el siguiente ejemplo.

r 

peso del cuerpo en el aire peso del cuerpo en el aire  peso del cuerpo en el agua

Ejemplo 12.6. Una corona de 14,7kg tiene un peso aparente cuando está sumergida en agua correspondiente a 13,4 kg. ¿Es de oro? Solución El eso aparente el objeto sumergido, W’ es igual a su peso real W menos la fuerza de empuje:

W '  W  FB  0 g V   f g V donde V es el volumen del objeto, ρ0 la densidad y ρf la densidad del fluido (agua en este caso). Entonces podemos escribir:

 g V 0 W  0  W W '  f gV  f

[Así, (W/W-W’) es igual a la densidad relativa del objeto si el fluido en el que esta sumergido es agua]. Para la corona tenemos:

0 14,7 kg W 14,7     11,3  H O W  W ' (14,7  13,4)kg 1,3 2

esto corresponde a una densidad de 11 300 kg/m3. ¡La corona parece estar hecha de plomo! El principio de Arquímedes se aplica con la misma validez a los objetos que flotan, como la madera. En general, un objeto flota en un fluido si su densidad es menor que la del fluido. Por ejemplo, un tronco cuya densidad específica es de 0,60 y cuyo volumen es 2,0 m3 tendrá una masa de 1 200 kg. Si está completamente sumergida, desplazará una masa de agua m = ρV = (1000 kg/m3)(2,0m3) = 2000 kg. De ahí que la fuerza de empuje sobre él será mayor que su peso y flotará hacia la superficie. Alcanzará el equilibrio cuando desplace 1200 kg de agua, lo que significa que 1,2 m3 o 0,60 de su volumen está sumergido. En general, la fracción del objeto sumergido está dada por la razón de la densidad del objeto a la del fluido. Ejemplo 12-6. Un hidrómetro es un instrumento utilizado para medir la densidad relativa. Un hidrómetro particular (Figura 12-11) consta de un tubo de vidrio, lastrado en la base, de 25,0 cm de largo, 2,00 cm2 de área de sección transversal y 45,0 g de masa. ¿A qué distancia del extremo debiera ponerse la marca de 1,000? Solución El hidrómetro tiene una densidad:

22

APUNTES DE FISICA

MsC.



JESUS ROBERTO GAVIDIA IBERICO

45,0 g m m    0,900 g / cm3 2 V A h 2,00 cm 25,0 cm





Así, cuando se coloca agua, llega al equilibrio cuando 0,900 de su volumen está sumergido. Como tiene una área de sección transversal uniforma, (0,900)(25,0 cm)= 22,5 cm de longitud estará sumergida. Como la densidad relativa del agua se define como 1,000, la marca debería ponerse a 22,5 cm del extremo.

Figura 12-11. Hidrómetro

El aire es un fluido y también ejerce una fuerza de empuje. Los objetos comunes pesan menos en el aire que cuando están en el vacío. Debido a que la densidad del aire es muy pequeña, el efecto para los sólidos comunes es apenas perceptible (pero véase los problemas al final del capitulo). Sin embargo, existen ciertos objetos que flotan en el aire, por ejemplo, los globos de helio. Ejemplo 12.7. ¿Qué volumen de helio se requiere si debe elevarse un globo con una carga de 800 kg (incluido el peso del globo vacío)? Solución La fuerza de empuje sobre el helio, FB, que es igual al peso del aire desplazado, debe ser cuando menos igual al peso del helio más la carga:

FB  mHe  800kg g

donde g es la aceleración debida a la gravedad. Esta ecuación puede escribirse en términos de la densidad:

aireg Vaire   He V  800kg g Resolviendo ahora para V, encontramos:

V

800kg 800kg 800 kg    720 m3 3 3  aire   He 1,29 kg / m  0,18 kg / m 1,11 kg / m3





12.7. TENSION SUPERFICIAL Hasta ahora en este capitulo hemos estudiado principalmente lo que ocurre debajo de la superficie de un liquido (o gas). Pero la superficie real de un líquido también se comporta de manera interesante. Numerosas observaciones sugieren que la superficie de un líquido actúa como una membrana estirada bajo tensión. Por ejemplo, una gota de agua sobre el extremo de una llave que gotea, o que pende de una rama delgada en el amanecer, toma una forma casi esférica como si fuera un pequeñísimo balón lleno de agua; una aguja de acero puede hacerse flotar en la superficie del agua aun cuando su densidad sea mayor que la de ésta. La superficie de un líquido actúa de ese modo bajo tensión, y ésta, que actúa paralela a la superficie, proviene de las fuerzas

APUNTES DE FISICA

MsC.

JESUS ROBERTO GAVIDIA IBERICO

23

atractivas entre las moléculas. Este efecto se llama tensión superficial. De manera más específica se define una cantidad llamada tensión superficial, γ (letra griega gamma), como la fuerza F por unidad de longitud L que actúa a través de cualquier línea en una superficie, tendiente a estirar la superficie abierta:



F L

(12-7)

Para entender lo anterior, considérese el aparato en forma de U mostrada en la Figura 12-12 que encierra una delgada película de liquido; debido a la tensión superficial, se necesita una fuerza F para jalar el alambre movible y así incrementar el área de la superficie del liquido. El líquido que contiene al aparato de alambre es una delgada película que tiene una cara anterior y otra posterior; de ahí que la longitud de la superficie que se incrementa es 2ℓ.

Figura 12-12. Aparato de alambre en forma de U sosteniendo una película de liquido para medir la tensión superficial (γ = F/2l)

De este modo la tensión superficial:



F 2l

Un aparato fino de este tipo puede usarse para medir la tensión superficial de distintos líquidos. La tensión superficial del agua es de 0,072 N/m a 20º C. En la Tabla 12-3 se proporcionan los valores para otros líquidos; nótese que la temperatura tiene un efecto considerable sobre la tensión superficial. Tabla 12-3. Tensión superficial de algunas sustancias Sustancia Tensión superficial (N/m) Mercurio (20º C) 0,44 Sangre (37º C) 0,058 Plasma sanguíneo (37º C) 0,073 Alcohol etílico (20º C) 0,023 Agua (0o C) 0,076 Agua (20º C) 0,072 Agua (100º C) 0,059 Benceno (20º C) 0,029 Solución jabonosa (20º C) 0,025 Oxigeno (- 193º C) 0,016 Podemos observar de dónde procede la tensión superficial al examinar el proceso desde el nivel molecular. Las moléculas de un liquido ejercen fuerzas atractivas entre si; estas fuerzas se muestran en acción en la Figura 1213, sobre una molécula sumergida en un liquido y sobre otra que está en la superficie. La primera está en equilibrio debido a las fuerzas de otras moléculas que actúan en todas direcciones. La molécula que está en la superficie está asimismo generalmente en equilibrio (el liquido se encuentra en reposo); esto se cumple aun cuando las fuerzas sobre una molécula en la superficie sólo pueden ejercerlas las moléculas que están debajo de ella (o a la misma altura). De ahí que hay una fuerza neta de atracción hacia abajo, que tiende a comprimir la capa superficial ligeramente [pero sólo hasta el punto en que esta fuerza hacia abajo se equilibra con otra fuerza ascendente (de repulsión) debida a un contacto cercano (o colisión con) las moléculas inferiores]. Esta compresión de la superficie significa que el líquido trata de reducir su área superficial al máximo. Esta es la razón por la que el agua tiende a formar gotas esféricas, porque una esfera representa el área superficial mínima para un volumen dado.

24

APUNTES DE FISICA

MsC.

JESUS ROBERTO GAVIDIA IBERICO

Figura 12-13. Teoría molecular de la tensión superficial, en donde se muestran las fuerzas atractivas (solamente) sobre una molécula en la superficie y sobre otra sumergida en el liquido.

Con el fin de incrementar el área superficial de un líquido se necesita una fuerza y debe realizarse un trabajo para llevar moléculas del interior a la superficie (Figura 12-14). Este trabajo incrementa la energía potencial de las moléculas y se denomina en ocasiones energía superficial. Mientras mayor sea el área superficial, mayor será la energía de superficie. La cantidad de trabajo necesario para incrementar el área superficial puede calcularse de la figura 12-12 y de la ecuación 12-7:

W  F x   L x   A

donde Δx es el cambio en la distancia y ΔA el incremento total en el área (en ambas superficies de la figura 1212) y de ese modo podemos escribir:



W A

Así, la tensión superficial γ no sólo es igual a la fuerza por unidad de longitud; sino también es igual al trabajo hecho por unidad de incremento de área superficial. De ahí que γ pueda especificarse en N/m o en J/m2.

Figura 12-14. Fuerzas sobre una molécula llevada a la superficie debido un incremento del área superficial. Las moléculas en la superficie deben ejercer una gran fuerza sobre la molécula transportada hacia arriba y a su vez experimentan una gran fuerza de reacción (tercera ley de newton). Esta es la fuerza de la tensión superficial

Debido a la tensión superficial, los insectos pueden caminar sobre el agua y los objetos más densos que ésta, como una aguja de acero, pueden flotar realmente sobre la superficie. La figura 12-15a muestra cómo puede soportar el peso W de un objeto la tensión superficial. En realidad, W es el “peso efectivo” del objeto (su peso verdadero menos la fuerza de empuje) puesto que el objeto se sumerge ligeramente en el fluido. Si el objeto tiene forma esférica, que es aproximadamente la forma que tienen las patas de los insectos (véase la figura 1215b), la tensión superficial actúa en todos los puntos a lo largo de un circulo de radio r. Sólo la componente vertical, γ cosθ, actúa para equilibrar W. En consecuencia, la fuerza neta ascendente debida a la tensión superficial es 2π r γ cosθ.

Figura 12-15. Tensión superficial actuando sobre (a) una esfera, (b) la pata de un insecto

Ejemplo 12-8. La base de la pata de un insecto tiene una forma más o menos esférica con un radio de 2,0 x 10-5 m. La masa del insecto es de 0,0030 g y está soportada de igual manera por las seis patas. Calcule el ángulo θ (Véase la Figura 12-15). Suponga que la temperatura del agua es de 20º C. Solución

APUNTES DE FISICA

Como:

MsC.

JESUS ROBERTO GAVIDIA IBERICO

25

F  0 2 r  cos  W

donde W es la sexta parte del peso del insecto (dado que éste tiene seis patas) tenemos:

6,28322,0 x105 m0,072 N / mcos  1 3,0 x106 kg9,8 m / s 2  6

cos 

0,49  0,54 0,90

  570 Nótese que si θ fuera mayor que 1, indicaría que la tensión superficial no seria lo suficientemente grande para soportar el peso. Los cálculos como el anterior no siempre son preciso pues el radio r de la superficie comprimida no es exactamente igual al radio del objeto. Sin embargo, pueden efectuarse cálculos aproximados para determinar si un objeto permanecerá sobre la superficie o no. Los jabones y los detergentes desminuyen la tensión superficial del agua. Este efecto es deseable para lavar y limpiar, pues la elevada tensión superficial del agua pura le impide penetrar con facilidad en las fibras de distintos materiales y en pequeñas cantidades. Las sustancias que disminuyen la tensión superficial de un líquido se llaman surfactantes. 12.8. CAPILARIDAD Se puede observar sin dificultad que el agua de un recipiente de vidrio se eleva ligeramente cuando toca el vidrio, Figura 12-16a . Se dice que el agua “moja” el vaso. El mercurio, en cambio, se sume cuando toca el vaso, figura 12-16b; el mercurio no moja el vaso. El hecho de que un líquido moje o no una superficie sólida se determinan mediante los esfuerzos relativos de las fuerzas de cohesión entre las moléculas del líquido en comparación con las fuerzas de adhesión entre las moléculas del líquido y del sólido. (La cohesión se refiere a la fuerza entre las moléculas del mismo tipo y la adhesión a las fuerzas entre las moléculas distintas). El agua moja el vaso debido a que sus moléculas son atraídas con más fuerza por las moléculas del vidrio que por las demás moléculas de agua. El caso contrario ocurre con el mercurio; las fuerzas de cohesión son mayores que las de adhesión.

Figura 12-16. El agua (a) “moja” la superficie del vaso, en tanto que el mercurio (b) no.

El ángulo que la tangente a la superficie del liquido forma con al superficie sólida se llama el ángulo de contacto . Su valor depende de los esfuerzos relativos de las fuerzas de cohesión y de adhesión (véase la Figura 12-16 y la tabla 12-4). Cuando  es menor de 90º, el líquido moja el sólido; si  es mayor de 90º, no.

26

APUNTES DE FISICA

MsC.

JESUS ROBERTO GAVIDIA IBERICO

Tabla 12-4. Angulo de contacto de algunas sustancias Sustancia Agua –Vidrio Líquidos orgánicos(la mayor parte)-vidrio Mercurio-vidrio Agua – parafina Keroseno – vidrio

Angulo de contacto 00 00 140º 107º 26º

FIGURA 12-17. Capilaridad.

En tubos que tienen diámetros muy pequeños se observa que los líquidos se elevan o se hunden en relación con el nivel del líquido de los alrededores. Este fenómeno se conoce por capilaridad y dichos tubos delgados se llaman capilares. El que un líquido suba o baje (Fig. 12-17) depende de los esfuerzos relativos de las fuerzas adhesivas y cohesivas. Así, el agua sube en un tubo de vidrio en tanto que el mercurio baja. La cantidad real que sube (o que baja) depende de la tensión superficial (puesto que es ésta la que mantiene unida a la superficie del líquido), así como del ángulo de contacto  y el radio r del tubo. Para calcular h, la altura que sube, nos referiremos a la figura 12-18. La tensión superficial  actúa en un ángulo  alrededor de un círculo de radio r. La magnitud de la fuerza vertical F debida a la tensión superficial es, mediante la ecuación 12-7, F = ( cos )(L). Como L = 2r, tenemos F = 2  r  cos . Esta fuerza está equilibrada en forma exacta por el peso del líquido de abajo que es aproximadamente un cilindro de altura h y volumen V = r2h. En consecuencia:

2 r  cos  m g   V g    r 2 g donde  es la densidad del líquido. Resolviendo para h encontramos:

h

2  cos   gr

(12-8)

Para la mayor parte de los líquidos como el agua en un vaso,  es casi cero y como cos 0° = 1, la ecuación 12-8 se convierte simplemente en:

h

2  gr

La ecuación 12-8 también se cumple cuando desciende el líquido, como en el caso del mercurio en un tubo de vidrio. En esta situación, el ángulo de contacto , es mayor que 90° y cos  será negativo; esto hace h negativa lo que corresponde a un descenso de nivel. Nótese de acuerdo a la ecuación 12-8 que mientras más delgado sea el tubo mayor será el ascenso (o descenso) del líquido.

APUNTES DE FISICA

MsC.

JESUS ROBERTO GAVIDIA IBERICO

27

FIGURA 12-18. Un líquido en un tubo capilar se eleva una altura h dada por h = 2 cos /gr

EJEMPLO 12-9 Si el xilema (pequeños tubos que llevan hacia arriba los nutrientes) de una planta tiene un radio de 0.0010 cm, calcule qué tan alto puede la tensión superficial jalar una columna de agua. Suponga que  = 0°. SOLUCION Con  = 0,072 N/m en la ecuación 12-8, encontramos:

h 12.9.

2  cos  2 (0,072 N / m) (cos 0o )   1,5 m  gr (1,0 x103 kg / m3 ) (9,80 m / s 2 ) (1,0 x105 m)

PRESION NEGATIVA Y COHESION DEL AGUA

Por lo regular, un fluido ejerce una presión hacia afuera sobre su recipiente y éste ejerce una presión hacia adentro sobre el fluido. Las presiones pueden variar desde cero hasta un valor positivo muy alto. Pero, ¿puede ser negativa la presión del fluido? La respuesta, por extraño que parezca es afirmativa; en ciertas condiciones puede haber una presión negativa en un líquido pero no así en un gas.

FIGURA 12-19. Forma de provocar una presión negativa.

Las presiones negativas se han obtenido, aunque no sin dificultad, usando el aparato mostrado en la figura 1219. El tubo cerrado en un principio está lleno con líquido y entonces la presión en el recipiente de la derecha se reduce con una bomba de vacío. De acuerdo con la ecuación 12-6a, la diferencia de presión entre los puntos A y B es gh, donde  es la densidad del líquido:

PB  PA   g h Cuando la bomba ha reducido la presión hasta un valor cercano a cero, PB = 0, dado que el punto B se encuentra al mismo nivel que la superficie del líquido del recipiente. De ahí que la presión en A deba ser negativa:

PA   g h En general, podríamos esperar que el líquido descendiera del tubo cuando la bomba disminuye la presión. Pero si el tubo se limpia escrupulosamente y el líquido no contiene impurezas, puede permanecer en el tubo.

28

APUNTES DE FISICA

MsC.

JESUS ROBERTO GAVIDIA IBERICO

Presiones negativas de hasta  270 atmósferas se han obtenido por medio de esta técnica. El líquido bajo presión negativa es inestable y la menor perturbación hará que se esparza en pequeñas gotas y descienda. Un líquido sometido a presión negativa puede en realidad jalar hacia dentro sobre su recipiente; el líquido actúa como si todo estuviera bajo tensión (no sólo su superficie) de manera semejante a una cuerda que se estira por sus dos extremos. ¿Cómo puede ocurrir? Esto se debe a las fuerzas de cohesión entre las moléculas del líquido así como a las fuerzas de adhesión entre el líquido y el recipiente. Las fuerzas de cohesión entre las moléculas del agua son muy fuertes (el esfuerzo tensor del agua pura es de unos 30 x 106 N/m2). Estas son las mismas fuerzas que mantienen unidas a las moléculas de agua en forma de hielo a bajas temperaturas. La diferencia es que en estado líquido las moléculas tienen energía cinética considerable y son libres de moverse una sobre otra. El agua líquida normalmente no sostiene una fuerza de tensión como lo hace un sólido debido a que la más ligera intromisión de una impureza (como el aire) permite al agua fluir y tomar otra forma; en ese caso puede descender por gravedad. Durante mucho tiempo se ha considerado la cuestión de cómo puede subir el agua a las copas de los árboles altos como en los árboles que llegan alcanzar alturas superiores a los 100 m. Como se vio en el ejemplo 12-9, incluso los capilares más pequeños pueden levantar agua sólo 1.5 m. Por otra parte, con vacío, la presión atmosférica puede elevar el agua sólo unos 10 m incluso cuando el extremo superior se encuentre al vacío (Sección 12-4). Ninguno de estos efectos participa para elevar el agua en los árboles altos. Actualmente se acepta que aunque los efectos mencionados arriba pueden ser útiles, son las fuerzas de cohesión entre las moléculas de agua que actúan bajo tensión a presiones negativas las que son responsables, en particular para los árboles muy altos. Esta teoría fue propuesta por vez primera el siglo pasado, pero hasta hace poco tiempo tenía muchos detractores. Durante los últimos años se han efectuado mediciones reales de la presión dentro del xilema de los árboles; se encontró de manera inequívoca que es negativa, con frecuencia menor que 25 atm en las copas. Cuando el agua se evapora por las hojas, otras moléculas de agua se mueven para ocupar su lugar. A través de las fuerzas cohesivas el agua de abajo es jalada hacia arriba.

RESUMEN Los tres estados de la materia son sólidos, líquidos y gaseosos. Los líquidos y los gases se llaman fluidos, debido a su capacidad para fluir. La densidad de una sustancia se define como su masa por unidad de volumen. La densidad relativa de una sustancia es la razón de su densidad a la del agua (a 4°C). La presión se define como fuerza por unidad de área. Un fluido en reposo ejerce una presión igual en todas direcciones en cualquier punto de su interior. A una profundidad h debajo de la superficie de un líquido de densidad uniforme, , la presión debida al líquido es igual a gh donde g es la aceleración de la gravedad. Si la densidad de un fluido no es uniforme, la presión P varía con la altura y como dp/dy = g. Si se aplica una presión externa a la superficie de un fluido confinado (como es el caso de la presión atmosférica sobre un recipiente abierto o una masa de agua), ésta se transmite íntegramente a través del fluido; esto se conoce como principio de Pascal. La presión se mide con un manómetro u otro tipo de medidores de presión. Para medir la presión atmosférica se utiliza el barómetro. La presión atmosférica estándar al nivel del mar es 1.013 x 105 N/m2. La presión manométrica es la presión total menos la atmosférica. El principio de Arquímedes establece que un objeto sumergido total o parcialmente en un fluido es empujado hacia arriba por una fuerza igual al peso del fluido que se desaloja. Este principio se usa en los métodos para determinar el peso específico así como explicar por qué los objetos cuya densidad es menor que la de un fluido flotan en él. PREGUNTAS 1.

¿Cuál es el peso específico de (a) el aire, (b) el hielo, (c) el oro?

2.

Si un material tiene más densidad que otro, ¿significa eso que las moléculas del primero deben ser más pesadas que las del segundo? Explíquelo.

3.

Desarrolle un método sencillo para calcular la densidad de su cuerpo usando la alberca.

APUNTES DE FISICA

MsC.

JESUS ROBERTO GAVIDIA IBERICO

29

4.

Los aeropasajeros notan con frecuencia que sus frascos de cosméticos y otros recipientes se han fugado después del viaje. ¿Qué puede provocar esto?

5.

Los tres recipientes de la figura 12-20 están llenos con agua hasta una misma altura y tienen la misma área de superficie en la base; en consecuencia, la presión del agua y la fuerza total sobre la base de cada uno es la misma. Aun así el peso total del agua es diferente en cada uno. Explique por qué esta "paradoja hidrostática" no es contradictoria.

FIGURA 12-20

6.

Considere qué ocurre cuando empuja un alfiler y una barra contra su piel con igual fuerza. Decida qué determina que su piel se corte: la fuerza neta aplicada o la presión.

7.

Un corcho flota en una botella de agua abierta a la atmósfera. Si el lector usa una bomba para comprimir el aire en la botella, ¿podría hundir al corcho? Explíquelo.

8.

Desarrolle un método para determinar la masa de una de sus piernas usando una alberca.

9.

Describa cómo se hace el lavado del oro, con base en los principios de este capítulo.

10.

¿Por qué flota usted con más facilidad en agua salada que en agua dulce?

11.

¿Es la fuerza de empuje sobre una campana de buzo sumergida profundamente en el océano la misma que la que tiene cuando está sólo abajo de la superficie? Explíquelo.

12.

¿Subirá un globo indefinidamente en el aire?

13.

Una barcaza cargada con piedras se aproxima a un puente de baja altura sobre el río y no puede pasar por debajo. ¿Debieran añadirse o quitarse piedras a la barcaza?

14.

Es más difícil jalar el tapón del desagüe de una tina cuando ésta está llena de agua que cuando está vacía. ¿Es ésta una contradicción al principio de Arquímedes? Explíquelo.

15.

Un cubo de hielo flota en un vaso de agua lleno hasta el borde. Cuando se derrita el hielo, ¿se derramará el vaso?

16.

Explique por qué un lápiz de madera uniforme siempre flota en forma horizontal en vez de hacerlo vertical. Explique por qué puede flotar verticalmente si se pone un lastre suficiente a uno de sus extremos.

17.

Una jarra de agua es acelerada de manera uniforme hacia la derecha a lo largo de una superficie horizontal. Explique por qué la superficie del agua forma un ángulo con la horizontal. En qué dirección se dirige la pendiente de la superficie?

18.

(a) Muestre que la fuerza de empuje sobre un objeto parcialmente sumergido actúa en el centro de gravedad del fluido antes que sea desplazado. Este punto se llama centro de flotación. (b) Para que un barco tenga estabilidad, su centro de flotación deberá estar encima, abajo o en el mismo lugar que su centro de gravedad? Explíquelo.

19.

¿Tendrá un globo vacío exactamente el mismo peso en una báscula que otro lleno con aire? Explíquelo.

20.

Un globo flotante lleno de helio está amarrado mediante una cuerda pequeña al asiento de un auto. Si el auto da una vuelta a la izquierda, ¿cómo se moverá el globo con relación al auto?

21.

¿Por qué un barco que zozobra se vuelca sobre un lado antes de hundirse? (Sugerencia: véase la pregunta 18.)

30

APUNTES DE FISICA

MsC.

JESUS ROBERTO GAVIDIA IBERICO

22.

Es válido el principio de Arquímedes en un elevador que tiene una aceleración (a) de ½ g, (b) de caída libre?

23.

Una pieza de madera está sumergida 60 por ciento en un recipiente de agua sobre la Tierra. ¿flotará o se hundirá si el recipiente está en un elevador con una aceleración de ½ g (a) hacia arriba, (b) hacia abajo? (c) ¿Qué ocurre cuando el elevador está en caída libre?

24.

Una pequeña cantidad de agua se pone a hervir en una lata de gasolina de 1 galón. La lata se quita del fuego y se tapa. Poco, después; la lata se colapsa. Explíquelo.

25.

Se dice a menudo que "el agua busca su nivel". Explíquelo.

26.

¿Hay un límite de profundidad a la que un buzo puede usar un snorkel debajo del mar? Explíquelo.

27.

Explique como el tubo de la figura 12-21, conocido como sifón, puede transferir líquido de un contenedor a uno inferior aun cuando el líquido tenga para ello que subir un trecho. (Nótese que el tubo debe llenarse con líquido para comenzar.) ¿Por qué el líquido de cada lado del tubo no regresa a su recipiente?

FIGURA 12-21 Un sifón.

28.

Aproximadamente, ¿a qué altura esperaría que estuviera la columna de mercurio de un barómetro usado en un satélite en órbita terrestre a una altura de 6400 km?

29.

Cuando se mide la presión sanguínea, ¿por qué la manga unida al manómetro se coloca sobre su brazo a la altura del corazón?

30.

¿Por qué el marco de una ventana y/o del alféizar tiene una ranura semejante a la de la figura 12-22?

31.

Un pato puede flotar en el agua debido a que arregla sus plumas para aplicar una cubierta de grasa. Explique cómo la tensión superficial incrementada le permite flotar al pato.

FIGURA 12-22

PROBLEMAS SECCION 12-1. 12-1.

El volumen aproximado de un monolito de granito conocido como "El Capitán" en el Parque Nacional de Yosemite es de unos 108 m3. ¿Cuál es su masa aproximada?

RESPUESTA: 2,7 x 1011 kg 12-2.

¿Cuál es la masa aproximada del aire en una estancia de 6.8 m x 3.4 m x 2.8 m?

APUNTES DE FISICA

12-3.

MsC.

JESUS ROBERTO GAVIDIA IBERICO

31

Si se añaden 5.5 L de una solución anticongelante (DR = 0.80) a 4.5 L de agua para hacer 10.0 L de mezcla, ¿cuál es la densidad relativa de esa mezcla?

RESPUESTA: 0,89 12-4.

Una botella tiene una masa de 31.20 g cuando está vacía y de 98.44 g cuando está llena con agua. Cuando se llena con otro fluido, su masa es de 88.78 g. ¿Cuál es el peso específico del fluido?

SECCION12-2 12-5.

El brazo de un tocadiscos ejerce una fuerza de (1.0 g) x g sobre un disco. Si el diámetro de la aguja es de 0.0013 cm ( = 0.5 mil = 0.5 x 103 in), calcule la presión sobre el surco del disco en N/m2 y en atmósferas.

RESPUESTA: 7,4 x 107 N/m2; 740 atm 12-6.

¿Cuál es la diferencia en la presión sanguínea entre la parte superior de la cabeza y la planta de los pies en una persona de 1.60 m de alto en posición vertical?

12-7.

¿Cuál es la diferencia aproximada en la presión del aire entre la azotea y la base del World Trade Center Building en Nueva York, que tiene 410 m de altura y se localiza al nivel del mar? Exprésela como una fracción de la presión atmosférica al nivel del mar.

RESPUESTA: 0,052 12-8.

Cuando maneja cuesta arriba en las montañas o desciende rápidamente por ellas, sus oídos pueden hacer un chasquido, lo que significa que la presión detrás del tímpano está igualándose con la exterior. Si no ocurriera eso, ¿cuál sería la fuerza aproximada sobre un tímpano de 0.50 cm2 de área si ocurriera un cambio de altura de 1000 m?

12-9.

Muestre que el trabajo realizado cuando actúa una presión constante P para mover un volumen de fluido V es W = P V.

12-10.

Calcule la presión en la cima del monte Everest (8850 m sobre el nivel del mar).

12-11.

¿Cuál es la fuerza total sobre un dique rectangular de 75 m de altura y 120 m de ancho si se llena de agua hasta el tope?

RESPUESTA: 3,3 x 109 N 12-12.

Un brazo de un tubo en forma de U (abierto por ambos lados) contiene agua y el otro alcohol. Si los dos fluidos se juntan exactamente en la base del tubo en U y el alcohol se encuentra a una altura de 18.0 cm, ¿a qué altura está el agua?

12-13.

Calcule la masa total de la atmósfera terrestre usando el valor conocido de la presión atmosférica al nivel del mar.

RESPUESTA: 5,3 x 1018 kg 12-14.

Estime la densidad del agua a 10.0 km de profundidad en el mar. (Véase la Sección 11-4 y la Tabla 111.) ¿En qué fracción difiere de su densidad en la superficie?

12-15.

Determine la presión general a una profundidad h en un fluido de densidad  si el fluido y su recipiente tienen una aceleración a que ocurre (a) hacia arriba, (b) hacia abajo, (c) igual a g hacia abajo (caída libre).

RESPUESTA: (a) P0 +  (g + a) h; (b) P0 + (g  a) h; (c) P0

32 12-16.

APUNTES DE FISICA

JESUS ROBERTO GAVIDIA IBERICO

Una cubeta cilíndrica de líquido (densidad ) se rota alrededor de su eje de simetría que está en posición vertical. Si la velocidad angular es , muestre que la presión a una distancia r del eje de rotación es P  Pa 

12-17.

MsC.

1   2 r 2 donde Pa es la presión en el eje a la misma profundidad. 2

En la base de un tubo en forma de U cuyos dos extremos están abiertos a la atmósfera hay agua pura a iguales niveles. Por un lado del tubo en forma de U se vacía un segundo líquido que no se mezcla con el agua. En el otro lado del tubo el agua se eleva 8.3 cm y su superficie está 2.1 cm encima de la del segundo líquido. ¿Cuál es la densidad de este último líquido?

RESPUESTA: 1,15 x 103 kg/m3 12-18.

Una jarra de líquido se acelera desde el reposo sobre una superficie horizontal con una aceleración a hacia la derecha. (a) Muestre que la superficie del líquido forma un ángulo  = tan1 a/g con la horizontal (b) ¿Qué lado de la superficie del agua está más alto? (c) ¿Cómo varía la presión con la profundidad debajo de la superficie?

12-19.

Detrás de un dique vertical de anchura uniforme b hay agua hasta una altura h. (a) Use una integral para mostrar que la fuerza total del agua sobre el dique es FT  1/ 2  g h 2 b . (b) Muestre que la torca alrededor de la base del dique debida a esta fuerza se puede considerar que actúa con un brazo de palanca igual a h/3. (c) Para un dique de concreto autoestable de grosor uniforme ℓ y altura h, ¿qué grosor mínimo se necesita para evitar que se derrumbe? ¿Necesita añadir la presión atmosférica en esta parte? Explíquelo.

SECCIONES 12-3 y 12-4 12-20.

Un valor típico para la presión sistólica sanguínea es 120 mmHg. Convierta esto a (a) torr, (b) N/m2, (c) atm, (d) lb/pulg2.

12-21.

(a) Calcule la fuerza total sobre la atmósfera que actúa sobre la cubierta de una mesa cuyas dimensiones son 3.2 m x 1.2 m. (b) ¿Cuál es la fuerza total que actúa hacia arriba sobre el lado de abajo de la mesa?

RESPUESTA: (a) 3,9 x 105 N hacia abajo; (b) 3,9 x 105 N hacia arriba 12-22.

Determine la presión manométrica mínima que se necesita en la tubería de agua que surte a un edificio, si el agua debe salir de una llave del décimo segundo piso a 40 m.

12-23.

La contracción del ventrículo izquierdo (cámara) del corazón bombea sangre al cuerpo. Suponiendo que la superficie interior del ventrículo izquierdo tiene un área de 85 cm2 y que la presión máxima en la sangre es 120 mmHg, calcule la fuerza neta ejercida por el ventrículo en el punto de presión máxima.

RESPUESTA: 1,4 x 102 N 12-24.

Suponga que una persona puede reducir la presión en los pulmones hasta  80 mmHg de presión manométrica. ¿A qué altura puede entonces ser aspirada el agua en un popote (cañita de plástico)?

12-25.

Un manómetro de mercurio de tubo abierto se usa para medir la presión en un tanque de oxígeno. En un día en que la presión atmosférica es 1040 mbars, ¿cuál es la presión absoluta (en Pa) en el tanque si la altura del mercurio en el tubo abierto es: (a) 28.0 cm de altura, (b) 4.2 cm más bajo que el mercurio en el tubo conectado al tanque?

RESPUESTA: (a) 1,41 x 105 N/m2; (b) 0,98 x 105 N/m2 12-26.

Las infusiones intravenosas con frecuencia se ponen conectando la aguja (insertada en el brazo) mediante un tubo a una botella elevada llena de líquido. Suponiendo que éste tiene una densidad de 1.00 g/cm3, ¿a qué altura h deberá colocarse la botella de modo que la presión del líquido sea (a) 60

APUNTES DE FISICA

MsC.

JESUS ROBERTO GAVIDIA IBERICO

33

mmHg, (b) 600 mmH20? (c) Si la presión de la sangre se encuentra 18 mmHg encima de la atmosférica, ¿a qué altura deberá colocarse la botella de modo que el fluido apenas entre a la vena? Sea h la altura vertical de la superficie del fluido en la botella encima de la aguja. 12-27.

La presión manométrica en cada una de las cuatro llantas de un auto de 1800 kg es de 210 kN/m2. ¿Qué área de la llanta está en contacto con el suelo?

RESPUESTA: 140 cm2 12-28.

Durante cada latido, aproximadamente 70 cm3 de sangre son bombeados desde el corazón a una presión promedio de 105 mmHg. Calcule la potencia de salida del corazón en watts, suponiendo que hay 60 latidos por minuto.

12-29.

¿A qué altura deberá estar el nivel en un barómetro de alcohol a una presión atmosférica normal?

RESPUESTA: 13 m SECCION 12-5 12-30.

La presión manométrica en un gato hidráulico es de 16 atm. ¿Cuál es el vehículo más grande (kg) que puede levantar si el diámetro de la línea de salida es de 17 cm?

12-31.

Se aplica una fuerza de 3.0 N al émbolo de una aguja hipodérmica. Si el diámetro del émbolo es 1.0 cm y el de la aguja es de 0.20 mm. (a) ¿Con qué fuerza saldrá el fluido por la aguja? (b) ¿Qué fuerza sobre el émbolo se necesitaría para empujar el fluido dentro de una vena donde la presión manométrica es 18 mmHg?

RESPUESTA: (a) 1,2 x 103 N; (b) 0,19 N 12-32.

Para elaborar su principio, Pascal mostró en forma tajante cómo podía multiplicarse la fuerza con la presión de un fluido. Colocó verticalmente un tubo largo de 0.30 cm de radio en un barril de vino de 20 cm de radio. Encontró que cuando el barril se había llenado con agua y el tubo estaba lleno hasta una altura de 12 m, el barril se reventaba. Calcule (a) la masa del fluido en el tubo, (b) la fuerza neta sobre la tapa del barril.

SECCION 12-6 12-33.

El hidrómetro del ejemplo 12-6 se hunde a una profundidad de 22.3 cm cuando se coloca en un tanque cervecero. ¿Cuál es la densidad de la cerveza?

RESPUESTA: 1,01 x 103 kg/m3 12-34.

Un geólogo encuentra que una piedra lunar cuya masa es de 7.20 kg tiene una masa aparente de 5.88 kg cuando está sumergida en agua. ¿Cuál es la densidad de la piedra?

12-35.

¿Cuánto mide la fuerza de empuje debida a la atmósfera sobre un tanque de agua de 4 700 m3?

RESPUESTA: 5,94 x 104 N 12-36.

¿Qué porción de una pieza de hierro se sumergirá cuando esté flotando en mercurio?

12-37.

Una pieza de madera de 0.40 kg flota en agua pero se encuentra que se hunde en alcohol (DR = 0.79), en el que tiene una masa aparente de 0.020 kg. ¿Cuál es la densidad relativa de la madera?

RESPUESTA: 0,83 12-38.

Use la tabla 12-1 para determinar qué fracción de un cubo de hielo (puro) se sumerge en un vaso de (a) agua pura, (b) agua salada. (c) ¿Cuáles serian sus respuestas si el experimento se llevara a cabo en la

34

APUNTES DE FISICA

MsC.

JESUS ROBERTO GAVIDIA IBERICO

Luna donde g es un sexto de su valor en la Tierra? (d) ¿Se aplicaría la respuesta a alguno de los incisos (a) o (b) a un iceberg en el océano? De ser así, ¿a cuál? 12-39.

Una cubeta de agua se acelera hacia arriba a 3.5 g. ¿Cuál es la fuerza de empuje sobre una piedra de granito de 1.0 kg (DR = 2.7) sumergida en el agua? ¿Flotará la piedra? ¿Por qué?

RESPUESTA: 16 N; No 12-40.

Un cubo de madera está sumergido 50 % en un lago. ¿Qué trabajo se requiere para sumergirlo?

12-41.

Un buque de carga tiene un área de sección transversal horizontal de 3100 m2 en la línea de flotación. Cuando se carga, el buque se hunde 6.1 m. ¿Cuál es la masa de la carga?

RESPUESTA: 1,9 x 107 kg 12-42.

Cuál es1a DR de un animal de 25 kg si, cuando flota en agua, sólo 2.0 cm3 permanecen encima de la superficie?

12-43.

Se observa que un pequeño objeto permanece suspendido en una mezcla de 18 % de alcohol (en peso) y 82 % de agua. ¿Cuál es su densidad?

RESPUESTA: 0,95 x 103 kg/m3 12-44.

Un bote de remos tiene un volumen de 1.5 m3 y una masa de 35 kg. ¿Cuántas personas, cada una de 70 kg de masa, puede transportar el bote sin hundirse.

12-45.

El principio de Arquímedes puede usarse no sólo para determinar el peso específico de un sólido usando un líquido conocido (Ejemplo 12-5); el caso contrario también es factible. (a) Como ejemplo, considere una bola de aluminio de 12.00 kg que tiene una masa aparente de 9.40 kg cuando está sumergida en un líquido particular; calcule la densidad del líquido. (b) Derive una fórmula simple para determinar la densidad de un líquido usando este procedimiento.

RESPUESTA: (a) 585 kg/m3; (b)

l  1  

m'   m

12-46.

Calcule la masa real (en el vacío) de una pieza de aluminio cuya masa aparente es 2, 0000 kg cuando se pesa en el aire.

12-47.

Una pieza de madera de 520 g (DR = 0.50) flota sobre el agua. ¿Qué masa mínima de plomo, que penda de ella por medio de una cuerda, la hará hundirse?

RESPUESTA: 0,57 kg 12-48.

Si un objeto flota en el agua, su densidad puede determinarse atándole un "hundidor" de modo que el objeto y el peso se hundan. Muestre que el peso específico está dado por W/(W1  W2) donde W es el peso del objeto sólo en el aire, W1 el peso aparente del objeto cuando se le ata un "hundidor" y sólo éste se sumerge, y W2 el peso aparente del objeto cuando tanto el objeto como el hundidor están sumergidos.

12-49.

Dos cubos sólidos de madera, cada uno de volumen 1.00 m3, tienen pesos específicos de 0.35 y 0.60. Las caras de área igual se ponen juntas, se nivelan y se colocan. Si la combinación se coloca luego en forma horizontal (los dos bloques lado a lado) en agua de manera que flote, ¿qué torca neta actúa sobre ésta, tratando de ponerla vertical?

RESPUESTA: 1,2 x 103 N · m SECCION 12-7

APUNTES DE FISICA

MsC.

JESUS ROBERTO GAVIDIA IBERICO

35

12-50.

Si la fuerza F necesaria para mover el alambre en la figura 12-12 es 6.4 x 103 N, calcule la tensión superficial y del fluido encerrado. Suponga que ℓ = 0.075 m.

12-51.

La tensión superficial de un líquido puede determinarse midiendo la fuerza F necesaria para apenas levantar un anillo circular de platino de radio r de la superficie del líquido. (a) Encuentre una fórmula para  en términos de F y r. (b) A 30° C, si F = 9.40 x 103 N y r = 3.5 cm, calcule  para el líquido probado.

RESPUESTA: (a) F/4r; (b) 2,1 x 102 N/m 12-52.

¿Cuánto trabajo se necesita para incrementar el tamaño de una burbuja de jabón (véase la Tabla 12-3) de 3.0 cm diámetro a 5.0 cm de diámetro?

12-53.

Un pequeño charco de agua sobre una mesa se parte en 50 gotas. ¿En qué factor cambia la energía de superficie? Suponga que originalmente el charco es aplanado y tiene una profundidad h y que las gotas son semiesféricas de radio h.

RESPUESTA: 3,0 x (despreciando las superficies de la base) 12-54.

Muestre que en el interior de una burbuja de jabón debe haber una presión P que excede a la exterior igual a P = 4 / R donde R es el radio de la burbuja y  es la tensión superficial. (Sugerencia: suponga la burbuja como dos hemisferios en contacto entre sí; recuerde que hay dos superficies en la burbuja. Nótese que este resultado se aplica a cualquier clase de membrana donde T = 2  es la tensión por unidad de longitud en la membrana.)

SECCION 12-8 12-55.

¿Cuánto subirá el agua en un tubo de vidrio de 0.12 mm de radio?

RESPUESTA: 0,12 m 12-56.

Un tubo de vidrio de 0.85 mm de diámetro se pone en mercurio. ¿Dónde estará el nivel de mercurio en el tubo con relación al resto del líquido?

12-57.

Cuando se pone un tubo de vidrio en un recipiente de alcohol etílico, el líquido asciende 3.4 mm por el tubo. ¿Cuál es el diámetro del tubo?

RESPUESTA: 1,7 mm 12-58.

Un lápiz de 1.0 cm de diámetro se sostiene en forma vertical en un vaso de agua. El agua moja el lápiz de manera que el ángulo de contacto es 0°. Calcule la magnitud y dirección de la fuerza neta sobre el lápiz debido a la tensión superficial.

12-59.

¿Qué tan delgados deberían ser los tubos del xilema si el agua tuviera que subir hasta la copa de un árbol de 100 m de altura sólo mediante la acción de la capilaridad?

RESPUESTA: 1,5 x 107 m (20 ºC) 12-60.

¿Cuánto se elevará el agua debido a la capilaridad entre dos placas planas de vidrio separadas 0.11 mm cuando se colocan en agua en forma vertical?

36

APUNTES DE FISICA

MsC.

JESUS ROBERTO GAVIDIA IBERICO

CAPITULO 13 DINAMICA DE FLUIDOS: FUIDOS EN MOVIMIENTO INTRODUCCIÓN Ahora pasamos del estudio de los fluidos en reposo al estudio más complicado de los fluidos en movimiento, que se conoce como hidrodinámica. Muchos aspectos del movimiento de los fluidos todavía no se entienden por completo; aún así, adoptando algunas simplificaciones, puede obtenerse una buena comprensión de esta materia. Una aproximación al estudio del flujo de los fluidos consiste en seguir partículas individuales (o pequeños elementos de volumen) del fluido. El movimiento de cada partícula, como se rige por las leyes de Newton, podría calcularse en principio, pero esto resulta en extremo complicado y difícil. En vez de eso, la aproximación más usual que consideraremos aquí, es describir las propiedades del fluido en cada punto en el espacio. Esto es, en lugar de seguir el movimiento de cada partícula del fluido a medida que se mueve a través del espacio en función del tiempo, observaremos cada punto en el espacio y describiremos el movimiento del fluido (consignando la velocidad del fluido y su densidad) en cada punto en función del tiempo. 13.1 CARACTERÍSTICAS DEL FLUIDO Podemos distinguir dos tipos principales de fluido. Si el fluido es uniforme de modo que los estratos contiguos del mismo se deslicen entre si de manera continua, se dice que el flujo es una línea de corriente o flujo laminar. Este tipo de fluido se caracteriza por el hecho de que cada partícula del fluido sigue una trayectoria uniforme, y porque estas trayectorias no se cruzan entre si, figura 13.1a . Al rebasar cierta velocidad que depende de un gran número de factores, como veremos más tarde, el flujo se hace turbulento. El flujo turbulento se caracteriza por círculos pequeños de manera de remolinos, erráticos, llamados corrientes parásitas o remolinos, figura 13.1b. Las corrientes parásitas absorben una gran cantidad de energía y aunque cierta cantidad de fricción interna llamada viscosidad se presenta durante los flujos laminares, ésta es mucho mayor cuando el flujo es turbulento. Unas cuantas gotas de tinta o colorantes echadas en un liquido en movimiento pueden revelar de manera rápida si el flujo es laminar o turbulento.

Figura 13.1. (a) Líneas de corriente o flujo laminar, (b) flujo turbulento Tanto para los flujos laminares como turbulentos, podemos considerar cuatro características del flujo de fluidos: 1. El fluido puede considerarse compresible o incompresible; aunque ningún material es verdaderamente incompresible, el flujo de muchos fluidos es tal que las variaciones de densidad son tan pequeñas que pueden ignorarse, lo que simplifica en gran forma el análisis. 2. La viscosidad o fricción interna, siempre está presente en el movimiento del fluido, pero también es con frecuencia lo suficientemente pequeña para ignorarla; en la primera parte de este capitulo consideraremos flujos no viscosos y en secciones posteriores investigaremos los efectos de la viscosidad.

APUNTES DE FISICA

3.

MsC.

JESUS ROBERTO GAVIDIA IBERICO

37

El flujo puede ser estacionario, lo que significa que la velocidad del fluido en cada punto en el espacio permanece constante en el tiempo (lo que no necesariamente implica que la velocidad sea la misma en todos los puntos en el espacio). Si la velocidad en un punto cambia en el tiempo (como cuando el agua comienza a moverse en un tubo en el momento en que se abre una llave) el flujo es no estacionario; nos ocuparemos de manera principal de los flujos estacionarios.

4. El flujo puede ser rotacional o irrotacional. Es irrotacional si no hay un momento angular neto del fluido en cada punto; esto es, una pequeña rueda de paletas colocada en cualquier lugar del fluido no rotaria; si la rueda rotara, como en un remolino o corriente parásita, el fluido seria rotacional. Nos ocuparemos aquí en forma directa de esta característica más bien complicada. 13.2 GASTO Y ECUACIÓN DE CONTINUIDAD En el fluido laminar estacionario de un fluido, la trayectoria seguida por una partícula dada se llama flujo laminar (véase la Figura 13.1 a). la velocidad del fluido en cualquier punto es tangente al flujo laminar en ese punto. Un flujo laminar puede, en principio, dibujarse a través de cada punto del fluido, aún cuando normalmente nosotros dibujamos sólo unas cuantas líneas. Dos líneas de flujo no pueden intersectarse; puesto que esto supondría que en el punto de intersección la velocidad no estaría bien definida. Un haz de líneas de flujo, como las que se muestran en la Figura 13.2, se llaman tubos de flujo. Como las líneas de flujo representan las intersecciones de partículas, vemos que ningún fluido puede fluir hacia adentro o hacia fuera de los lados de un tubo de flujo.

Figura 13.2. Un tubo de flujo

A continuación vamos a estudiar el flujo laminar estacionario de un tubo de flujo y determinar cómo la rapidez del fluido varia con el tamaño del tubo. Escogeremos el tubo suficientemente pequeño para que la velocidad a través de cualquier sección transversal sea, en esencia constante. Así, en la figura 13.2, v1 representa la velocidad cuando pasa a través del área de sección transversal A1 y v2 la velocidad cuando pasa a través del área de sección transversal A2. El gasto de masa se define como la masa Δm de fluido que pasa por un punto dado por unidad de tiempo Δt; gasto másico = Δm/Δt. En la Figura 13.2, el volumen de fluido que pasa por el punto 1 (es decir, a través del área A1) en el tiempo Δt es exactamente A1Δl1 donde Δl1 es la distancia que el fluido se mueve en el tiempo Δt. Como la velocidad del fluido que pasa en el punto 1 es v1 = Δl1/Δt, el gasto másico Δm/Δt a través del área A1 es (donde ΔV1 = A1Δl1 es el volumen de masa Δm):

Q

 m 1 V1 1 A1 l1    1 A1 v1 t t t

de manera similar en el punto 2 (a través del área A2), el gasto es ρ2A2v2. Como ningún fluido entra o sale de los lados del tubo de flujo, el gasto a través de A1 y A2 debe ser el mismo. Así:

Q1  Q2

1 A1v1  2 A2v2 Esta se llama ecuación de continuidad. Si el flujo es incompresible, que es una excelente aproximación para los líquidos en la mayor parte de las circunstancias (y con frecuencia también para los gases), entonces ρ 1 = ρ2 y la ecuación de continuidad se convierte en:

A1v1  A2v2

[fluido incompresible]

(13.1)

38

APUNTES DE FISICA

MsC.

JESUS ROBERTO GAVIDIA IBERICO

Nótese que el producto Av representa el gasto volumétrico de flujo (volumen de fluido que pasa por un punto dado por segundo), puesto que ΔV/Δt = A Δl/Δt = Av. La ecuación 13,1 nos dice que donde el área de sección transversal de un tubo de flujo (o simplemente de un tubo) es grande, la velocidad es baja; y que donde el arrea es pequeña, la velocidad es alta. Que esto tenga sentido puede observarse viendo un río; un río fluye lenta y apaciblemente en las vegas, donde el cuero es ancho, pero su rapidez se hace torrencial cuando pasa a través de un paso angosto. Asimismo notamos que la ecuación 13.1 y de la Figura 13.2 que las líneas de flujo más juntas (punto 2) indican mayor rapidez del fluido y que las líneas más espaciadas (punto 1) indican una rapidez de flujo menor. La ecuación 13.1 puede aplicarse al flujo sanguíneo del cuerpo, La sangre fluye del corazón a la aorta, de la que asa a las arterias mayores; éstas se ramifican en las arterias menores (arteriolas), que a su vez se ramifican en miríadas de pequeños vasos capilares; la sangre regresa al corazón a través de las venas. Ejemplo 13.1. El radio de la aorta es de aproximadamente 1,0 cm y la sangre que fluye a través de ésta tiene una rapidez de unos 30 cm/s. calcule la rapidez promedio de la sangre en los capilares valiéndose del hecho de que aun cuando cada capilar tiene un diámetro de unos 8 x 10-4 cm, existen literalmente miles de millones de ellos, de manera que su área de sección transversal total es de unos 2000 cm2. Solución Por la ecuación de continuidad:

A2v2  A1v1

La rapidez de la sangre en los capilares es:

v2 





A1 v1  r12 v1  1 x102 m 0,30 m / s     5 x104 m / s 1 2 A2 A2 2 x10 m 2

Otro ejemplo que utiliza la ecuación de continuidad y el argumento que conduce a ésta es el siguiente: Ejemplo 13.2. ¿Qué tamaño un ducto de calentamiento si el aire que se mueve a lo largo del mismo a 3,0 m/s puede reenlazar el aire de un cuarto de 300 m3 de volumen cada 15 minutos? Suponga que la densidad del aire permanece constante. Solución Consideremos el cuarto como una gran sección del ducto para propósitos de aplicación de la ecuación 13.1 (llamado punto 2). Razonando en el mismo sentido se obtuvo la ecuación 13.1. (Cambiando Δt por t), vemos que:

Q  A2 v2 

A2 l2 V2  t t

donde V2 es el volumen del cuarto. Entonces A1v1 = A2v2 = V2 / t y:

V2 / t V2 300m3 A1     0,11 m2 v1 v1 t 3,0 m / s 900 s  Como A = πr2 encontramos que el radio debe ser:

r

13-3

A





0,11m2



 0,19 m

ECUACIÓN DE BERNOULLI

¿Alguna vez se ha preguntado cómo puede circular el aire en la madriguera de una marmota, por qué sube el humo en una chimenea o por qué el techo de un auto convertible se comba hacia arriba a altas velocidades?

APUNTES DE FISICA

MsC.

JESUS ROBERTO GAVIDIA IBERICO

39

Estos son ejemplos de un principio deducido por Daniel Bernoulli (1700-1782) en los albores del siglo XVIII. En esencia, el principio de Bernoulli establece que donde la velocidad de un fluido es alta, la presión es baja y donde la velocidad es baja la presión es alta. Por ejemplo, si las presiones en los puntos 1 y 2 de la figura 13-2 se midieran, se encontraría que la presión es menor en el punto 2, donde la velocidad es mayor, que en el punto 1, donde la velocidad es menor. A primera vista, esto podría parecer extraño; usted podría esperar que la mayor rapidez en el punto 2 ocasionaría una presión mayor. Pero esto no puede ocurrir; pues si la presión en el punto 2 fuera mayor que en 1, esta presión mayor detendría el fluido, mientras que en realidad éste se ha acelerado. Así la presión en 2 debe ser menor que en 1 lo que permitirá que el fluido se acelere.

FIGURA 13-3. Flujo de fluidos: para la derivación de la ecuación de Bernoulli

Bernoulli formuló una ecuación que expresa este principio en forma cuantitativa. Para derivar la ecuación de Bernoulli, su póngase que el flujo es laminar y estacionario, que el fluido es incompresible y que la viscosidad es lo suficientemente pequeña para ignorarla. De manera general, consideraremos un tubo de flujo que varía (a lo largo de la longitud del tubo) en sección transversal así como en altura sobre un nivel de referencia, figura 133. Consideraremos la cantidad de fluido mostrado más oscuro y calcularemos el trabajo hecho para moverlo de la posición mostrada en (a) a la mostrada en (b). En este proceso el fluido en el punto 1 fluye una distancia ℓ1 y fuerza al fluido en el punto 2 a moverse una distancia ℓ2. El fluido a la izquierda del punto 1 ejerce una presión P1 sobre el fluido y realiza una cantidad de trabajo W1 = F1 ℓ1 = P1 A1 ℓ1. En el punto 2, el trabajo hecho en W2 = P2 A2 ℓ2; el signo negativo aparece porque la fuerza ejercida sobre el fluido se opone al movimiento (así el fluido mostrado (más oscuro) trabaja sobre el fluido a la derecha del punto 2). Asimismo, se hace un trabajo en el fluido por medio de la fuerza de gravedad; como el efecto neto del proceso mostrado en la figura 13-3 es mover una masa m de volumen A1ℓ1 (= A2 ℓ2) del punto 1 al punto 2, el trabajo hecho por la gravedad es:

W3   m g ( y2  y1 ) Nótese que en el caso mostrado en la figura 13-3 este término es negativo puesto que el movimiento es hacia arriba contra la fuerza de gravedad. El trabajo neto W realizado sobre el fluido es por tanto:

W  W1  W2  W3

W  P1 A1 1  P2 A2  2  mg y2  m g y1 De acuerdo con el teorema del trabajo y la energía (Sección 6-4), el trabajo neto realizado sobre un sistema es igual a su cambio en energía cinética. Así:

1 2 1 2 m v  m v  P1 A1  1  P2 A2  2  m g y2  m g y1 2 2 2 1 La masa m tiene un volumen A1 ℓ1 = A2 ℓ2 y de este modo podemos sustituir m =  A1 ℓl = A2 ℓ2 y obtener (después de dividir entre Al ℓ1( = A2 ℓ2 y reordenar términos):

1 1 P1   v12   g y1  P2   v22   g y2 2 2

(13-2)

Esta es la ecuación de Bernoulli. Como los puntos 1 y 2 pueden ser cualesquiera dos puntos a lo largo de un tubo de flujo, la ecuación de Bernoulli puede escribirse como:

40

APUNTES DE FISICA

MsC.

JESUS ROBERTO GAVIDIA IBERICO

1 P   v 2   g y  constan te 2 en todos los puntos del fluido.

FIGURA 13-4. Teorema de Torricelli.

La ecuación de Bernoulli puede aplicarse a una gran cantidad de casos. Un ejemplo de esto es calcular la velocidad, v1 de un líquido que sale de un grifo en la base de un recipiente, figura 13-4. Escogemos el punto 2 en la ecuación 13-2 como la superficie superior del líquido; suponiendo que el diámetro del recipiente es grande comparado con el de la llave, v2 será casi cero. Las presiones en los puntos 1 (la llave) y 2 son ambas iguales a la presión atmosférica de manera que P1 = P2, Entonces la ecuación de Bernoulli se convierte en:

1 2  v   g y1   g y2 2 1 o bien

v1  2 g ( y2  y1 )

(13-3)

Este resultado se llama teorema de Torricelli. Aunque se observa que es un caso especial de la ecuación de Bernoulli, fue descubierto un siglo antes que Bernoulli por Evangelista Torricelli (1608-1647), de ahí su nombre. Nótese que el líquido deja la llave con la misma velocidad con la que caería un objeto en caída libre de la misma altura. Esto no debe ser sorprendente ya que la derivación de la ecuación de Bernoulli descansa en la conservación de la energía. Otro caso especial de la ecuación de Bernoulli surge cuando el fluido se mueve sin que haya cambio apreciable en la altura; esto es, y1 = y2. Entonces la ecuación 13-2 se convierte en:

1 1 P1   v12  P2   v22 2 2

(13-4)

Esto nos dice de manera cuantitativa que donde la velocidad es alta la presión es baja, y viceversa. Esto explica muchos fenómenos cotidianos, algunos de los cuales se ilustran en la figura 13-5. La presión en el aire soplado a alta velocidad a través de la parte superior del tubo vertical de un atomizador de perfume (Fig. 13-5a) es menor que la presión normal del aire que actúa sobre la superficie del líquido en el frasco; así el perfume es empujado hacia arriba del tubo debido a la presión reducida en la parte superior. Una pelota de ping-pong puede hacerse flotar sobre un chorro de aire (algunas aspiradoras pueden soplar aire), figura 13-5b; si la pelota comienza a dejar el chorro de aire, la presión más alta de afuera del chorro empuja la pelota de nuevo hacia éste.

APUNTES DE FISICA

MsC.

JESUS ROBERTO GAVIDIA IBERICO

41

FIGURA 13-5. Ejemplos del principio de Bernoulli.

Las alas de los aviones y otros planos aerodinámicos se diseñan para desviar el aire de manera que aunque el flujo laminar se mantiene en gran medida, las líneas de flujo se aglomeran encima del ala, figura l3-5c. Así como las líneas de flujo se aglomeran en una construcción tubular donde la velocidad es alta, las líneas de flujo aglutinadas encima del ala indican que la velocidad del aire es mayor que debajo del ala. De ahí que la presión encima de ésta es menor que la de abajo, en consecuencia, hay allí una fuerza ascendente neta, ésta se llama sustentación dinámica. En realidad, el principio de Bernoulli es sólo un aspecto de la sustentación de una ala. Las alas se inclinan un poco hacia arriba, de modo que el aire que choca contra la superficie inferior se desvíe hacia abajo; el cambio en el momento de las moléculas de aire que rebotan deviene en una fuerza ascendente adicional sobre el ala. De igual modo la turbulencia desempeña una función de gran importancia. Un velero puede moverse contra el viento; figura l3-5d, y el efecto de Bernoulli ayuda a esto considerablemente si se arreglan las velas de modo que la velocidad del aire aumente en la angosta constricción entre las dos velas. (La presión normal detrás de la vela principal es mayor que la presión reducida en frente de ésta y la que empuja el bote hacia adelante.) Cuando se navega contra el viento, la vela principal se coloca en un ángulo aproximadamente a la mitad entre la dirección del viento y el eje del bote (línea de la quilla) como se muestra. La fuerza del viento sobre la vela (cambio de momento del viento que rebota de la vela), junto con el efecto de Bernoulli, actúa en forma casi perpendicular a la vela (Fviento). Esto podría tender a hacer que el bote se moviera hacia los lados, pero la quilla de abajo lo evita (debido a que el agua ejerce una fuerza (Fagua) sobre la quilla casi perpendicular a ésta). La resultante de estas dos fuerzas (FR) se dirige casi directamente hacia adelante, como se muestra. Un tubo de Venturi es básicamente un tubo con un angosto estrechamiento (la garganta). Un ejemplo de un tubo de Venturi es el barril de un carburador automotriz, figura l3-5e. El aire que fluye se acelera mientras pasa por este estrecho (Ecuación 13-1) y de ese modo la presión es menor. Debido a la reducción de la presión, la gasolina bajo la presión atmosférica en el recipiente del carburador se fuerza en la corriente del aire y se mezcla con el aire antes de entrar a los cilindros. El tubo de Venturi es también la base del venturímetro que se usa para medir la rapidez de flujo de los fluidos, figura 13-6. Puede mostrarse (véanse los problemas) que la velocidad de flujo está dada por la relación:

v1  A2

2 ( P1  P2 )  ( A12  A22 )

donde  es la densidad del fluido y P1 y P2 son las lecturas de la presión en los puntos 1 y 2 dónde el área del tubo es Al y A2. Si se usa el tipo manómetro (Fig. 13-6b) P1  P2 se sustituye por (m  ) gh donde m les la

42

APUNTES DE FISICA

MsC.

JESUS ROBERTO GAVIDIA IBERICO

densidad del fluido en el manómetro. Los venturímetros pueden usarse para medir las velocidades de flujo de los gases y de los líquidos e incluso se han diseñado algunos para medir la velocidad de la sangre en las arterias. La rapidez de flujo también puede medirse ya que es igual a v1Al, ¿Por qué sube el humo por una chimenea? En parte se debe a que el aire caliente se eleva (es decir, debido a la densidad). Pero el principio de Bernoulli también tiene un lugar importante. Debido a que el viento sopla a través de la parte superior de la chimenea, la presión es menor ahí que dentro de la casa. Por eso el aire y el humo son empujados hacia arriba de la chimenea. Incluso en una noche calmada, existe el flujo de aire suficiente en el ambiente en el extremo superior de la chimenea para permitir el flujo ascendente del humo.

FIGURA 13-6. Venturímetros: (a) estándar; (b) tipo manométrico.

Si las tuzas, perros de la pradera, conejos y otros animales que viven bajo el suelo no se asfixian, el aire debe circular en sus madrigueras. Estas siempre tienen al menos dos entradas. La velocidad del flujo del aire a través de los diferentes hoyos por lo regular será un poco distinta. Esto conduce a una pequeña diferencia de presión que fuerza al flujo de aire a través de la madriguera por el principio de Bernoulli. El flujo de aire se intensifica si un hoyo está más arriba que el otro (lo que a menudo hacen los animales) puesto que la velocidad del viento tiende a incrementarse con la altura. EJEMPLO 13-3 El agua circula a través de una casa, en un sistema de calefacción por agua caliente. Si se bombea el agua con una rapidez de 0.50 m/s por un tubo de 4.0 cm de diámetro en el sótano bajo una presión de 3.0 atm, ¿cuál será la rapidez de flujo y la presión en un tubo de 2.6 cm de diámetro ubicado en el segundo piso 5.0 m arriba? SOLUCION Primero calculamos la velocidad v2 usando la ecuación de continuidad, ecuación 13-1:

  R12   0,020 m  v1 A1   (0,50 m / s)    1,2 m / s v2   v1  2  A2  0,013 m    R2  2

Para encontrar la presión usamos la ecuación de Bernoulli:

1 P2  P1   (v12  v22 )   g ( y1  y2 ) 2





1 P2  (3,0 x105 N / m 2 )  (1,0x 103 kg / m3 ) (0,50 m / s) 2  (1,2 m / s) 2  2 3 3 (1,0 x10 kg / m )((9,8 m / s 2 ) (5,0 m)

P2  3,0 x105 N / m2  4,9 x104 N / m2  6,0 x 102 N / m2 P2  2,5 x 105 N / m2 Nótese que el término de la velocidad contribuye muy poco en este caso. La ecuación de Bernoulli no considera los efectos de fricción (viscosidad) y la compresibilidad del fluido. La energía que se transforma en energía interna (o potencial) debido a la compresión y a energía térmica debida a la fricción puede tomarse en cuenta agregando términos al lado derecho de la ecuación 13-2. Estos términos son

APUNTES DE FISICA

MsC.

JESUS ROBERTO GAVIDIA IBERICO

43

difíciles de calcular teóricamente y en general se determinan de manera empírica. No lo haremos aquí, sino que sólo señalamos que eso no cambia de modo significativo las explicaciones para el fenómeno descrito antes. 13-4

VISCOSIDAD

Como ya se mencionó, los fluidos reales tienen cierta cantidad de fricción interna que se llama viscosidad. Esta existe tanto en los líquidos como en los gases y en esencia es una fuerza de fricción entre diferentes capas de fluido que aparece cuando éstas pasan una sobre otra. En los líquidos se debe a las fuerzas de cohesión entre las moléculas; en los gases proviene de las colisiones entre las moléculas.

FIGURA 13-7. Determinación de la viscosidad.

Fluidos distintos poseen diferentes cantidades de viscosidad: el jarabe es más viscoso que el agua; la vaselina es más viscosa que el aceite para maquinaria; los líquidos en general son mucho más viscosos que los gases. La viscosidad de diferentes fluidos puede expresarse de manera cuantitativa mediante un coeficiente de viscosidad  (la letra minúscula griega eta), que se define de la siguiente manera. Una delgada capa del fluido se coloca entre dos placas planas. Una está inmóvil y la otra se mueve, figura 13-7. El fluido que se encuentra directamente en contacto con ambas placas es empujado hacia la superficie por las fuerzas adhesivas entre las moléculas del líquido y las de la placa. Así, la superficie superior del fluido se mueve con la misma velocidad v que la placa superior, en tanto que el fluido en contacto con la placa estacionaria permanece inmóvil. Este estrato de fluido retrasa el flujo del estrato que está justo encima; este estrato retrasa el flujo del siguiente y así sucesivamente. Por consiguiente, la velocidad varía en forma lineal de 0 a v como se muestra. El incremento en la velocidad dividido por la distancia sobre la cual se realiza este cambio (igual a v/ℓ) se llama el gradiente de velocidad. Para mover la placa superior se requiere una fuerza, un hecho que puede comprobarse moviendo una placa plana a través de un charco de jarabe encima de una mesa. Para un fluido dado, se encuentra que la fuerza necesaria, F, es proporcional al área de cualquiera de las placas, A, ya la velocidad, v, y que es inversamente proporcional a la separación de las placas, ℓ: F  vA/ℓ. Para diferentes fluidos, mientras más viscoso sea el fluido, mayor es la fuerza necesaria. De ahí que la constante de proporcionalidad para esta ecuación se define como el coeficiente de viscosidad, :

F  A Resolviendo para , encontramos:



v 

(13-5a)

F vA

Así las unidades del SI de  son N · s/m2 = Pa · s (pascal · segundo); en el sistema cgs la unidad es DINA · s/cm2 y esta unidad se llama poise (P). Las viscosidades se proporcionan con frecuencia en centipoises (cP), que es una centésima de un poise. En la tabla 13-1 se enumeran los coeficientes de viscosidad de diversos fluidos; asimismo se especifica la temperatura, dado que ésta tiene un marcado efecto (la viscosidad de líquidos como el aceite para máquinas, por ejemplo, decrece rápidamente cuando aumenta la temperatura). TABLA 13-1. Coeficientes de viscosidad para varios fluidos Fluido Agua

Sangre Plasma sanguíneo Alcohol etílico Aceite para maquinas Glicerina

Temperatura (oC) 0 20 100 37 37 20 30 20

Coeficiente de viscosidad, , (Pa · s) 1,8 x 103 1,0 x 103 0,3 x 103 4 x 103 1,5 x 103 1,2 x 103 200 x 103 1500 x 103

44

APUNTES DE FISICA

Aire Hidrogeno Vapor de agua *1 Pa · s = 10 P = 1 x 103 cP

MsC.

JESUS ROBERTO GAVIDIA IBERICO

0,018 x 103 0,009 x 103 0,013 x 103

20 0 100

La ecuación 13-5a se aplica cuando el gradiente de velocidad es uniforme. En el caso general de un gradiente de velocidad no uniforme, la ecuación 13-5a se convierte en:

F  A

dv dy

(13-5b)

donde el gradiente de velocidad dv/dy es la rapidez con que cambia la velocidad por unidad de distancia medida en forma perpendicular a la dirección de la velocidad. La proporcionalidad directa entre la fuerza y la velocidad dada por la ecuación 13-5a no se cumple para todos los fluidos (los fluidos para los que se cumple se denominan newtonianos). En forma alternativa, podemos decir que  es función de la velocidad para los fluidos no newtonianos; ejemplos de éstos son la sangre (que contiene corpúsculos) y otras suspensiones. 13-5

FLUJO LAMINAR EN TUBOS; ECUACIÓN DE POISEUILLE

Si un fluido no tuviera viscosidad, podría fluir a través de un tubo a nivel sin que se le aplicara fuerza alguna. Debido a la viscosidad se necesita una diferencia de presión entre los extremos de un tubo para que haya un flujo estacionario de cualquier fluido real, sea agua o aceite en un tubo, o sangre en el sistema circulatorio de un ser humano. La velocidad del flujo de un fluido en un tubo redondo depende de la viscosidad del fluido, de la diferencia de presión y de las dimensiones del tubo. El científico francés J. L. Poiseuille (1799-1869), que se interesó en la física de la circulación sanguínea (y por quien el "poise”, recibe ese nombre), determinó de qué manera afectan estas variables la velocidad del flujo de un fluido incompresible que sufre un flujo laminar en un tubo cilíndrico. Su resultado, conocido como la ecuación de Poiseuille, es el siguiente:

Q

 R 4 (P1  P2 ) 8 L

(13-6)

donde R es el radio interior del tubo, L su longitud, P1  P2 la diferencia de presión entre los extremos,  es la viscosidad, y Q el gasto volumétrico de fluido (el volumen de fluido que fluye por un punto dado por unidad de tiempo).

FIGURA 13-8. Fluido de densidad , viscosidad , fluyendo hacia la derecha en un tubo de radio R. Se muestra la variación en la velocidad del fluido a diferentes posiciones en el fluido.

FIGURA 13-9. Derivación de la ecuación de Poiseuille: (a) considérese un cilindro sólido de flujo con radio r; (b) considérese una coraza cilíndrica de radio r y espesor dr

APUNTES DE FISICA

MsC.

JESUS ROBERTO GAVIDIA IBERICO

45

A continuación derivaremos la ecuación de Poiseuille. Considérese un fluido sometido a un flujo laminar estacionario a través de un tubo cilíndrico de radio interior R como se muestra en la figura 13-8. Como un fluido tiende a adherirse a las paredes del tubo, cabe esperar que la velocidad del fluido será cero (o casi cero) en las paredes; en consecuencia, suponemos que la capa cilíndrica de fluido próxima a la pared del tubo tiene velocidad cero; cada capa consecutiva tiene una velocidad sólo un poco mayor debido a la fricción viscosa con la capa anterior. La velocidad se incrementa con la distancia a la pared y alcanza un máximo en el centro del tubo. Dicho gradiente de velocidad se indica en la figura 13-8. Primero determinamos v en función de r al considerar un cilindro sólido de fluido de radio r (r < R) cuyo eje se encuentra a lo largo del centro del tubo, como se muestra en la figura 13-9a. La fuerza sobre este cilindro debida a la diferencia en la presión en los extremos del tubo es:

F  P A  (P1  P2 ) A  (P1  P2 )  r 2

donde r2 es el área de los extremos de nuestro cilindro. El movimiento de este cilindro de fluido es retrasado por la fuerza viscosa ejercida por la siguiente capa de fluido que está justo fuera del cilindro; la magnitud de esta fuerza de viscosidad está dada por la ecuación 13-5b donde, para un área A, debemos usar el área de los lados del cilindro, A = (2r)(L). De este modo la fuerza de viscosidad es:

F   A

dv dv   (2 r L) dr dr

donde introducimos el signo negativo debido a que la fuerza se opone al movimiento. Como el fluido está sometido a un flujo estacionario, no hay aceleración. Por tanto, las dos fuerzas se equilibran:

(P1  P2 )  r 2  2  r L

dv dr

Resolvemos para el gradiente de velocidad y obtenemos:

(P  P ) r dv  1 2 dr 2 L Integramos esto para obtener v en función de r, la distancia desde el centro del tubo y notamos que v = 0 en r = R: v

d v   0

( P1  P2 ) r r dr 2 L R r

P  P r 2  v 1 2   2 L  2  R

v

P1  P2 2 2 (R  r ) 4 L

* La palabra laminar significa "en capas". Así que “flujo laminar" se refiere a nuestro modelo de flujo de fluido como llevándose a cabo en capas. Como se esperaba, la velocidad máxima se presenta en el centro del tubo (r = 0); su magnitud es proporcional al cuadrado del radio del tubo y asimismo proporcional al gradiente de presión, P/L = (P1  P2)/ L. Ahora que conocemos v en función de r, podemos determinar el gasto total de flujo Q a través del tubo, donde Q = dV/dt es el volumen total de fluido que pasa por una sección transversal del tubo por unidad de tiempo. Como la velocidad v no es constante a través del tubo, no podemos escribir simplemente Q = Av

46

APUNTES DE FISICA

MsC.

JESUS ROBERTO GAVIDIA IBERICO

(como en la Ecuación 13-1). En vez de eso, dividimos la sección transversal del tubo en pequeños anillos, de grueso dr, como se muestra en la figura 13-9b; luego calculamos el flujo a través de cada uno de estos anillos infinitesimales y los sumamos para obtener el gasto total, El área de los anillos infinitesimales mostrados en la figura 13-9b es el producto de su circunferencia, 2 r, por su ancho, dr:

d A  2 r d r Dado que la velocidad del flujo, v, depende sólo de r (Ecuación 13-7), v es la misma en todas partes del anillo infinitesimal. Así que, el gasto de fluido a través del anillo infinitesimal es:

d Q  v dA

dQ 

P1  P2 2 2 ( R  r ) 2 r d r 4 L

Integrando todos los anillos para obtener el gasto de flujo total á través del tubo: Q

 dQ  0

 (P1  P2 ) R 2 (R r  r 3 ) dr  2 L 0 R

 (P1  P2 )  R 2 r 2 r 4  Q   2 L  2 4 0 Q

 (P1  P2 ) R 4 8 L

que es la ecuación 13-6; de modo que hemos derivado la ecuación de Poiseuille a partir de la suposición del flujo laminar simple. EJEMPLO 13-4 El aceite para máquina (su póngase SAE 10-Tabla 13-1) pasa a través de un tubo delgado de 1.80 mm de diámetro en el prototipo de una máquina. El tubo tiene 5.5 cm de largo. ¿Qué diferencia de presión se necesita para mantener el gasto del flujo en 5.6 mL/min? SOLUCION El gasto en unidades del SI es Q = 5.6 x 106 m3/60 s = 9.3 x 108 m3/s. Resolvemos para P1  P2 en la ecuación 13-6 y ponemos todos los términos en unidades del SI:

P1  P2 

8 L Q 8 (2,0 x101 N ·s / m2 ) (5.5 x102 m) (9,3 x108 m3 / s)   4,0 x 103 N / m2 4 3 4  R  (0,90 x10 m)

o aproximadamente 0.04 atm. La ecuación de Poiseuille nos dice que el gasto Q es directamente proporcional al gradiente de presión (P1  P2)/L e inversamente proporcional a la viscosidad del fluido. Esto es exactamente lo que cabría esperar. Puede ser sorprendente, no obstante, que Q dependa también de la cuarta potencia del radio del tubo. Esto significa que para el mismo gradiente de presión, ¡si se duplica el radio del tubo el gasto se incrementa en un factor de dieciséis! De este modo el gasto, o de otra manera la presión requerida para mantener un gasto dado, es afectada en gran parte sólo por un pequeño cambio en el tubo del radio. Un ejemplo interesante de esta dependencia de R4 es el flujo de la sangre en el cuerpo humano. Sin embargo, como la ecuación de Poiseuille sólo se cumple para las corrientes laminares de un fluido incompresible con viscosidad constante , no puede calcularse con precisión para la sangre cuyo flujo ocurre no sin cierta turbulencia; la sangre contiene también corpúsculos (cuyo diámetro es casi igual al de los capilares) y 

APUNTES DE FISICA

MsC.

JESUS ROBERTO GAVIDIA IBERICO

47

depende hasta cierto punto de la velocidad del fluido v. No obstante, la ecuación de Poiseuille proporciona una primera aproximación. El cuerpo controla el flujo de sangre por medio de pequeñas bandas de músculo que rodean las arterias. La contracción de estos músculos reduce el diámetro de una arteria, que debido al factor R4 en la ecuación 13-6, el gasto se reduce en gran parte sólo con un pequeño cambio en el radio. De este modo acciones muy pequeñas de estos músculos pueden controlar en forma precisa el flujo de la sangre en las diferentes partes del cuerpo. Otro aspecto de esto es que el radio de las arterias disminuye como resultado de la arterioesclerosis (endurecimiento de las arterias) y por la formación del colesterol; cuando esto ocurre, el gradiente de presión debe incrementarse para mantener el mismo gasto. Si se reduce el radio a la mitad, el corazón tendrá que incrementar la presión en un factor de 16 para mantener el mismo gasto sanguíneo. El corazón debe trabajar más rápido en estas circunstancias pero por lo regular no puede mantener el gasto original. Así, la presión sanguínea elevada es un indicio tanto de que el corazón está trabajando más rápido como de que el gasto sanguíneo se ha reducido. 13-6

FLUJO TURBULENTO EN TUBOS; NÚMERO DE REYNOLDS

Si la velocidad del flujo es grande, el flujo a través de un tubo se convertirá en turbulento y no podrá aplicársele la ecuación de Poiseuille. Cuando el flujo es turbulento, el gasto Q para una diferencia de presión dada será menor que para un flujo laminar como está dado en la ecuación 13-6. Este es un resultado del hecho de que las fuerzas de fricción son mucho mayores cuando hay turbulencia. El impulso inicial de la turbulencia con frecuencia es abrupto y puede caracterizarse de manera aproximada mediante el denominado número de Reynolds, Re:

Re 

2 v  r 



(13-8)

donde v es la velocidad promedio* del fluido,  su densidad,  su viscosidad y r el radio del tubo en el que fluye el fluido. Los experimentos muestran que el flujo es laminar si Re tiene un valor menor que unos 2000 pero que es turbulento si Re sobrepasa este valor. * La velocidad media (promedio), r. se define como aquella velocidad uniforme sobre la sección transversal completa del tubo, que daría el mismo gasto de flujo volumétrico, Q. EJEMPLO 13-5 La velocidad promedio de la sangre en la aorta (r = 1.0 cm) durante la parte del ciclo del corazón en que descansa es de unos 30 cm/s. ¿El flujo es laminar o turbulento? SOLUCION Para responder esto calculamos el número de Reynolds usando los valores de  y  de las tablas 12-1 y 13-1:

Re 

2 v  r 



2 (0,30 m / s) (0,010 m)1,05 x103 kg / m3 )   1600 4,0 x103 N ·s / m2

Con probabilidad el flujo será laminar, pero cercano a la turbulencia. Nótese en este ejemplo dado que 1 N = 1 kg · m/s2, Re no tiene unidades. Por lo consiguiente vemos que el número de Reynolds siempre se presenta como una cantidad adimensional; su valor es el mismo en cualquier grupo consistente de unidades. 13-7 OBJETOS QUE SE MUEVEN EN UN FLUIDO; SEDIMENTACIÓN Y ARRASTRE En la sección anterior vimos cómo la viscosidad (y otros factores) afecta el flujo de un fluido a través de un tubo. En esta sección examinaremos un caso un poco diferente, la de un objeto que se mueve con relación a un fluido. Puede tratarse de un obstáculo que obstruya el flujo del fluido, como una gran piedra en un río o de un objeto que se mueve en un fluido, como un planeador o un auto que se mueven a través del aire, un submarino en el agua o una molécula que se sedimenta en una centrífuga.

48

APUNTES DE FISICA

MsC.

JESUS ROBERTO GAVIDIA IBERICO

Cuando un objeto se mueve con relación a un fluido, el fluido ejerce una fuerza sobre el objeto. Esta fuerza, que se conoce como fuerza de arrastre, se debe a la viscosidad del fluido y también, a altas velocidades, a la turbulencia detrás del objeto. Para caracterizar el movimiento de un objeto en relación con un fluido, es útil definir otro número de Reynolds

Re'

vL



(13-9)

donde  y  son la densidad y la viscosidad del fluido, v la velocidad del objeto relativa al fluido y L la longitud característica del objeto. Este número de Reynolds debe distinguirse claramente del utilizado para el gasto en un tubo (aún cuando su forma es semejante) puesto que los fenómenos son bien diferentes. Cuando el número de Reynolds para el presente caso es menor que alrededor de 1*, el flujo que rodea un objeto es en esencia laminar y se encuentra de manera experimental que la fuerza de viscosidad F, es directamente proporcional a la velocidad del objeto:

f  kv

(13-l0)

La magnitud de k depende del tamaño y forma del objeto y de la viscosidad del fluido. Para una esfera de radio r se ha calculado como: [esfera] k  6  r En consecuencia, la fuerza de viscosidad sobre una pequeña esfera, cuando el flujo es laminar, está dada por la ecuación de Stokes: [para una esfera] f  6  rv * Un objeto de 1 mm de largo se mueve a una velocidad de 1 mm/s a través de agua tiene un número de Reynolds igual a l. Esto mismo lo hace un objeto de 2 mm de largo que viaja en el aire a 7 mm/s. Entonces esta situación se aplica principalmente para pequeños objetos como gotas de lluvia, granos de polen y moléculas en una centrífuga. Para números de Reynolds mayores (normalmente por arriba de un valor entre 1 y 10), habrá turbulencia detrás del cuerpo conocida como estela (véase la Fig. 13-1b) y la fuerza de arrastre será mayor que la dada por la ecuación de Stokes para una esfera. Sin embargo, para objetos más aerodinámicos habrá menos turbulencia y de ahí que habrá menos arrastre. Cuando ocurre la turbulencia, los experimentos muestran que la fuerza de arrastre se incrementa como el cuadrado de la velocidad, f  v 2 . El incremento con la velocidad es entonces mucho más rápido que en el caso de un flujo estrictamente laminar. Cuando el número de Reynolds se aproxima a un valor cercano a 106, la fuerza de arrastre se incrementa en forma abrupta. Por encima de este valor la turbulencia existe no sólo detrás del objeto sino también en la capa de fluido que se encuentra próxima al cuerpo (llamada la capa de frontera) alrededor de todos sus lados. La sedimentación se refiere a los objetos pequeños que caen en un fluido, algunos ejemplos de esto son pequeñas partículas de piedra o minerales que sedimentan bajo el mar y los glóbulos rojos de la sangre que sedimentan en el fluido del plasma en un laboratorio.

FIGURA 13-10. Fuerzas sobre un objeto pequeño que caen a través de un fluido.

APUNTES DE FISICA

MsC.

JESUS ROBERTO GAVIDIA IBERICO

49

En un objeto de masa m que cae a través de un flujo bajo la acción de la gravedad se presentan numerosas fuerzas, como se muestra en la figura 13-10; la fuerza de la gravedad, mg; FB, la fuerza de empuje del fluido y Fv, la fuerza de viscosidad. Por la segunda ley de Newton, la fuerza neta es igual a la masa por la aceleración del objeto:

F  ma

m g  FB  Fv  m a La fuerza de empuje FB es igual al peso del fluido desplazado; esto es, FB = f Vg, donde f es la densidad del fluido, V el volumen del objeto (y por lo tanto el volumen desplazado) y g la aceleración debida a la gravedad. Podemos también escribir mg = 0 Vg donde 0 es la densidad del objeto. Usando la ecuación 13-10, podemos escribir la ecuación anterior como: (13-11) (  0   f )V g  k v  m a El primer término es el peso efectivo del objeto en el fluido. A medida que el objeto aumenta su velocidad, la fuerza de viscosidad crece hasta que se equilibra con el peso efectivo del objeto. En este punto la aceleración es cero y la velocidad ya no aumenta. Este valor máximo de vT, se llama la velocidad terminal o velocidad de sedimentación y se obtiene a partir de la ecuación 13-11 haciendo a = 0:

vT 

( 0   f ) V g k

(13-12)

La velocidad de sedimentación de los objetos pequeños como las macromoléculas y otras partes de las células es sumamente pequeña. Puede incrementarse usando una centrífuga (Capítulo 5) ya que el efecto de ésta es ejercer una fuerza sobre una partícula como si la aceleración de la gravedad se incrementara hasta un valor de 2r (véase la Ecuación 9-6); aquí  es la velocidad angular del rotor y r la distancia del objeto a partir del eje de rotación. De este modo, podemos todavía utilizar la ecuación 13-12 para una centrífuga si sustituimos la aceleración efectiva de la gravedad, igual a 2r, por g:

(  0   f )V  2 r vT  k

(13-13)

La centrifugación se usa con frecuencia para separar partículas o macromoléculas semejantes pero ligeramente diferentes (como dos tipos de ácidos nucleicos) así como para obtener información de gran importancia sobre el tamaño y la masa. RESUMEN El flujo de un fluido puede caracterizarse como laminar, en el que las capas del fluido se mueven de manera suave y regular a lo largo de las trayectorias llamadas líneas de flujo, o como turbulentos en el que el flujo es irregular y contiene remolinos. La viscosidad se refiere a la fricción dentro de un fluido y evita que éste fluya libremente; puede considerarse como una fuerza de fricción entre las diferentes capas del fluido a medida que éstas pasan una sobre otra. El gasto de un fluido es la masa o volumen de fluido que pasa por un punto dado por unidad de tiempo. Para un flujo laminar estable de un fluido en un tubo cerrado (no necesariamente uniforme), la ecuación de continuidad afirma que el gasto de masa, que es el producto de la densidad del fluido, la velocidad del flujo y el área de la sección transversal del tubo es la misma en todos los puntos del tubo: Av = constante. Si el fluido es incompresible, entonces Av = constante. El principio de Bernoullí establece que cuando la velocidad de un fluido es elevada, la presión es baja y que cuando la velocidad es baja, la presión es alta. En forma cuantitativa, la ecuación de Bernoulli para el flujo laminar estable de un fluido no viscoso e incompresible en un tubo de fluido (de sección transversal no uniforme y que varía con la altura) es:

APUNTES DE FISICA

50

MsC.

JESUS ROBERTO GAVIDIA IBERICO

1 1 P1   v12   g y1  P2   v22   g y2 2 2 para cualesquiera dos puntos a lo largo del tubo. Preguntas 1.

¿Los flujos laminares pueden ser tanto compresibles como incompresibles? ¿Viscosos o no viscosos? ¿Estacionarios o no estacionarios? ¿Rotacionales o irrotacionales? En caso de que la respuesta sea afirmativa dé un ejemplo. Explique su respuesta cuando sea negativa.

2.

¿Los flujos turbulentos pueden ser compresibles o incompresibles? ¿Viscosos o no viscosos? ¿estacionarios o no estacionarios? ¿Rotacionales o irrotacionales? En caso de que la respuesta sea afirmativa, dé un ejemplo. Explique su respuesta cuando sea negativa.

3.

¿Qué fuerzas intervienen para acelerar el fluido en la figura 13-2 cuando se mueve desde la región 1 hasta la región 2?

4.

¿Por qué el chorro de agua de una llave se hace más angosto cuando cae?

5.

Si se sostienen dos pedazos de papel en forma vertical separados unos centímetros (Fig. 13-11) y sopla entre ellos, ¿cómo cree que se moverá el papel? Hágalo y véalo. Explique qué sucede.

FIGURA 13-11

6.

Un lanzador de béisbol hace girar la pelota cuando lanza una curva. Usando el principio de Bernoulli, explique con cuidado por qué se curva la pelota. Explique por qué una pelota con superficie muy lisa al girar se curva en la dirección opuesta que otra con una superficie rugosa (como una pelota de béisbol o de tenis).

7.

¿Por qué los aviones despegan normalmente contra el viento?

8.

Se dice a los niños que eviten pararse muy cerca de un tren que se mueve muy rápido porque podrían ser succionados tras éste. ¿Es eso posible? Explíquelo.

9.

Los tejados de algunas casas son arrastrados en ocasiones durante un tornado o un huracán. Explique por qué ocurre esto usando el principio de Bernoulli

10.

¿Por qué se comba la cubierta de un convertible cuando el auto se mueve a toda velocidad?

11.

Dos barcos que se mueven en trayectorias paralelas cerca uno del otro corren el peligro de chocar. ¿Por qué?

12.

Explique por qué la rapidez del viento se incrementa cuando aumenta la altura sobre la superficie de la Tierra. . (Sugerencia: véase la Fig. 13-7.) Explique de qué modo afecta esto a un topo que construye su madriguera con una de sus entradas a mayor altura que la otra.

13.

Dos bolas de acero iguales se sueltan en tubos de agua a 10° ya 40 °C. ¿En cuál tubo llegará más rápido la bola al fondo?

APUNTES DE FISICA

MsC.

JESUS ROBERTO GAVIDIA IBERICO

51

14.

¿Por qué el aceite (o cualquier otro líquido) fluye más suave y rápidamente de una lata si se le abren dos agujeros (en lados opuestos de la tapa) en lugar de uno solo? ¿En qué condiciones se obtendrá un flujo continuo si sólo se abre un agujero?

15.

Los colibríes gastan 20 veces más energía en aletear frente a una flor que la que gastan en un vuelo normal. Explíquelo.

16.

Los glóbulos de la sangre tienden a flotar en el centro de los vasos sanguíneos. Explíquelo.

17.

Muestre que el flujo viscoso de un fluido en un tubo, figura 13-8, es rotacional.

PROBLEMAS SECCION 13-2 13-1.

Usando la información del ejemplo 13-1, calcule la velocidad promedio del flujo sanguíneo en las arterias mayores del cuerpo que tienen un área de sección transversal total de unos 2.0 cm2.

RESPUESTA: 47 cm/s 13-2.

Un ducto de aire de 15 cm de radio se usa para surtir un cuarto de 10 m x 5.1 m x 3.2 m cada 10 minutos ¿Cuál es la velocidad promedio del flujo del aire en el ducto?

13-3.

¿Cuánto tarda en llenarse una alberca de 9.5 m x 21.0 m cuya profundidad promedio es 3.1 m si el agua fluye desde un tubo de 1.9 cm de diámetro con una velocidad de 1.5 m/s?

RESPUESTA: 380 h 13-4.

La corriente de agua de una llave disminuye su diámetro a medida que cae. Derive una ecuación para el diámetro de la corriente en función de la distancia y bajo la llave, dado que el agua tiene una rapidez v0 cuando sale de la llave cuyo diámetro es D.

SECCION 13-3 13-5.

Muestre que la ecuación de Bernoulli se reduce a la variación hidrostática de la presión con la profundidad (Ecuación 12-6) cuando no hay ningún flujo (v1 = v2 = 0).

13-6.

¿Qué tan rápido fluye el agua de un agujero en la base de un tinaco de 4.6 m de profundidad lleno de agua? Ignore la viscosidad.

13-7.

¿Qué presión manométrica se requiere en una tubería si una manguera de bomberos debe surtir agua a una altura de 25 m?

RESPUESTA: 2,4 atm 13-8.

¿Qué carga de presión debe haber si el agua proviene de una llave con una velocidad de 8.0 m/s? Ignore la viscosidad.

13-9.

Si el viento sopla a 25 m/s sobre su casa, ¿cuál es la fuerza neta sobre ésta si su área es de 250 m2?

RESPUESTA: 1,0 x 105 N hacia arriba 13-10.

Si la presión absoluta del agua al nivel de la calle es 2.75 x 105 N/m2, ¿será suficiente para elevar agua a la azotea de un edificio de 50 m de altura? ¿Qué altura máxima puede tener el edificio?

13-11.

¿Cuál es el gasto volumétrico de agua en un llave de 1.8 cm de diámetro si la carga de presión ahí es de 10 m?

52

APUNTES DE FISICA

MsC.

JESUS ROBERTO GAVIDIA IBERICO

RESPUESTA: 3,6 x 103 m3/s 13-12.

¿Cuál es la sustentación (en newtons) debida al principio de Bernoulli sobre un ala con un área de 50 m2 si el aire pasa sobre las superficies superior e inferior con velocidad de 320 m/s y 290 m/s, respectivamente?

13-13.

Muestre que la potencia necesaria para conducir un fluido a través de un tubo es igual al gasto volumétrico, Q, por la diferencia de presión, P1  P2.

13-14.

El agua, a una presión de 3.3 atm al nivel de la calle, fluye en un edificio de oficinas a una velocidad de 0.50 m/s a través de una tubería de 5.0 cm de diámetro. La tubería se angosta hasta tener un diámetro de 2.5 cm en el último piso a 25 metros de altura. Calcule la velocidad del flujo y la presión en esta tubería del último piso. Ignore la viscosidad.

13-15.

Un aeroplano tiene una masa de 2.0 x 106 kg y el aire fluye bajo la superficie inferior de las alas a 100 m/s. Si éstas tienen un área superficial de 1200 m2, ¿qué tan rápido debe pasar el flujo sobre la superficie superior del ala si el avión debe permanecer en el aire? Considere sólo el efecto de Bernoulli.

RESPUESTA: 190 m/s 13-16.

Un motor de auto de cuatro cilindros tiene un desplazamiento de 2000 cm3; esto es aproximadamente el aire que es llevado dentro de los cuatro cilindros por cada revolución. Si el motor corre con una rapidez de 1500 rpm y el tubo de Venturi del carburador tiene un radio de 2.5 cm, (a) ¿Cuál es la rapidez del flujo del aire a través de éste? (b) ¿Cuál es la presión (en atmósferas) en el tubo de Venturi?

13-17.

En la figura 13-4, tome en cuenta la rapidez de la superficie superior de la tapa del tanque y muestre que la rapidez del fluido que sale por el agujero de la base es: v1  2 g h /(1  A12 / A22 ) donde h = y2  y1 y A1 y A2 son las áreas del agujero y de la superficie superior, respectivamente.

13-18.

Suponga que la superficie superior del vaso de la figura 13-4 está sujeto a una presión externa P0. (a) Derive una fórmula para la velocidad, v1 a la que fluye el líquido a partir del agujero en la base a la presión atmosférica, P0. Suponga que la velocidad de la superficie del líquido, v2, es aproximadamente cero. (b) Si P0 = 0.85 atm y y2  y1 = 2.1 m, determine v1 en el caso del agua.

13-19.

El tubo Pitot es un instrumento que se usa para medir la velocidad de un flujo, como se muestra en la figura 13-12. El fluido pasa el agujero en B con una rapidez v, mientras que en A el fluido se encuentra en reposo. El manómetro registra la diferencia de presión en estos dos puntos, PA  PB. (o) Derive una ecuación para la velocidad del flujo v en términos de la altura h, y la densidad r del fluido en movimiento y la densidad M del fluido en el manómetro. (b) Para medir el flujo de la sangre se usa un pequeño tubo Pitot; éste indica una altura h = 20 mmHg en el manómetro; calcule la velocidad del flujo sanguíneo.

Figura 13-12. Tubo d Pitot

RESPUESTA: (a) (2ghm / h )1/ 2 ; (b) 2,3 m/s

APUNTES DE FISICA

MsC.

JESUS ROBERTO GAVIDIA IBERICO

53

13-20.

Un tubo de 6.0 cm de diámetro se angosta gradualmente hasta 4.0 cm. Cuando el agua fluye a través de él a cierta velocidad, la presión mano métrica en estas dos secciones es 32 kPa y 24 kPa, respectivamente. ¿Cuál es el gasto volumétrico?

13-21.

Empuje de un cohete. (a) Use la ecuación de Bernoulli y la ecuación de continuidad para mostrar que la velocidad de emisión de los gases propulsores de un cohete es: v  2 (P  P0 ) /  ,donde  es la densidad del gas, P la presión del gas dentro del cohete y P0 la presión atmosférica en la vecindad del orificio de salida. Suponga que la densidad del gas permanece constante, que el área del orificio de salida, A0, es mucho menor que el área de sección transversal, A, dentro del cohete (considérelo como un cilindro) y que la rapidez del gas no es tan alta para que ocurra turbulencia o un flujo no estacionario. (b) Muestre que la fuerza de empuje sobre el cohete debida a los gases emitidos es

F  2 A0 (P  P0 )

13-22.

(a) Muestre que la velocidad del flujo medido por un medidor de Venturi está dada por la relación

v1  A2

2 (P1  P2 ) Vea la figura 13-6a. (b) Para el manómetro mostrado en la figura 13-6b,  ( A12  A22)

encuentre v1 en términos de h y otras cantidades relevantes. (c) Un tubo de Venturi mide el flujo de agua. Tiene un diámetro principal de 3.0 cm que se angosta hasta un diámetro de garganta de 1.0 cm. La diferencia de presión P1  P2 es de 18 mm Hg. Calcule la velocidad del fluido. 13-23.

Suponga que la abertura del tanque de la figura 13-4 está a una altura h1 sobre la base y que la superficie del líquido se encuentra a una altura h2, sobre la base. El tanque descansa sobre el nivel del suelo. (a) ¿A qué distancia horizontal de la base del tanque caerá el fluido al suelo? (b) ¿A qué otra altura, h1' puede hacerse un agujero para que el líquido que salga tenga el mismo "alcance"?

RESPUESTA: (a) 13-24.

h1 (h2  h1 ) ; (b) h2  h1

(a) En la figura 13-4, muestre que el nivel del líquido, h = y2  y1, disminuye con una rapidez

2 g h A12 dh  donde Al y A2 son las áreas del agujero y de la superficie superior e ignora la dt A22  A12 viscosidad. (b) Determine h en función del tiempo por integración. Sea h = h0 en t = 0. (c) ¿Cuánto tardará en vaciarse un cilindro de 9.4 cm de altura lleno con 1.0 L de agua si el agujero en la base tiene un diámetro de 0.50 cm? SECCION 13-4 13-25.

Un viscosímetro consta de dos cilindros concéntricos, de 10.20 cm y 10.60 cm de diámetro. Un liquido particular llena el espacio entre ellos a una profundidad de 12.0 cm. El cilindro exterior está fijo y una torca de 0.024 N · m mantiene al cilindro interior girando con una velocidad angular constante de 62 rev/min. ¿Cuál es la viscosidad del líquido?

RESPUESTA: 7,0 x 102 Pa · s 13-26.

(a) Muestre que el líquido viscoso de la figura 13-7 sufre un esfuerzo cortante en forma semejante a un sólido (Sección 11-4). (b) Muestre que la razón de cambio en el tiempo del esfuerzo cortante (cambio en el esfuerzo cortante por unidad de tiempo; véase la Sección 11-4) está dado por v/ℓ. (c) Muestre que el coeficiente de viscosidad está definido como:

 SECCION 13-5

esfuerzo cor tan te razón e cambio del esfuerzo cor tan te

54 13-27.

APUNTES DE FISICA

MsC.

JESUS ROBERTO GAVIDIA IBERICO

Calcule el gasto volumétrico de agua a través de un tubo de 15 m de longitud y 1.0 cm de diámetro si la diferencia de presión entre sus extremos es 0.35 atm y la temperatura es 20 °C.

RESPUESTA: 5,7 x 104 m3/s 13-28.

Un jardinero considera que le toma mucho tiempo regar un jardín con una manguera de 3/8 de pulgada. ¿En qué factor se reducirá el tiempo si usa una manguera de 5/8 de pulgada de diámetro? Suponga que las demás variables no cambian.

13-29.

Calcule la caída de presión por centímetro a lo largo de la aorta usando la información del ejemplo 131 y la tabla 13-1.

RESPUESTA: 0,96 Pa/cm 13-30.

Suponiendo un gradiente de presión constante, ¿en qué factor debe decrecer el radio de las venas si el flujo sanguíneo se reduce en un 80 por ciento?

13-31.

Un paciente recibe una transfusión de sangre. La sangre debe fluir a través de un tubo a partir de una botella elevada a una aguja insertada en la vena. El diámetro interior de ésta es 0.50 mm, su longitud 4.0 cm y el gasto requerido es de 4.0 cm3 de sangre por minuto. ¿A qué altura deberá colocarse la botella encima de la aguja? Obtenga  y  de las tablas 12-1 y 13-1. Suponga que la presión de la sangre es 20 torr más que la presión atmosférica.

RESPUESTA: 0,93 m 13-32.

(a) ¿Cuál debe ser la diferencia de presión entre los dos extremos de una sección de tubo de 2.0 km de 40 cm de diámetro si va a transportar petróleo ( = 950 kg/m3,  = 2.0 P) a una razón de 400 cm3/s? (b) ¿Con qué razón se produce la energía térmica en estas condiciones?

13-33.

¿Qué diámetro debe tener un ducto de aire si la presión de un sistema de ventilación y aire acondicionado debe rellenar el aire de una habitación de 10 m x 18 m x 4.0 m cada 10 min? Suponga que la bomba puede ejercer una presión manométrica de 4.0 x 104 atm

RESPUESTA: 16 cm 13-34.

Debe bombearse agua a través de un tubo de 10.0 cm de diámetro por una distancia de 300 m. El extremo más alejado del tubo se encuentra a 20 m encima de la bomba y a presión atmosférica normal. ¿Qué presión manométrica debe desarrollar la bomba para que exista algún flujo en la tubería?

13-35.

Muestre que la curva de v contra r, la velocidad del flujo laminar de un fluido viscoso en un tubo contra la distancia al centro del tubo, es una parábola (Fig. 13-8 y Ecuación 13-7).

13-36.

La velocidad del agua en el centro de un tubo de 5.2 cm de diámetro y 20 m de largo es 18 cm/s. Determine (a) la diferencia de presión entre los extremos, (b) el gasto volumétrico.

13-37.

Cuando un líquido viscoso fluye a través de un tubo circular, como en la figura 13-8, muestre que la velocidad promedio de flujo (la velocidad que si fuera uniforme a lo largo del tubo daría el mismo gasto) es igual a la mitad de la velocidad del fluido en el centro (la velocidad máxima).

13-38.

Un chorro de agua salta a 14.6 m de altura a partir de un tubo de 1.0 cm de diámetro en una fuente. ¿Cuál debe ser la presión en la bomba situada 4.2 m bajo el surtidor (en el suelo)? Tome en cuenta la viscosidad pero ignore la resistencia del aire. Anote cualquier simplificación que haga.

13-39.

Considere un sifón que pasa agua (20 °C) de un vaso a otro (más abajo) como se muestra en la figura 12-21. (a) Determine el gasto si la manguera tiene un diámetro de 1.2 cm y la diferencia en los niveles del agua de los dos contenedores es de 64 cm. (b) ¿Cuál es la altura máxima que la manguera puede alcanzar para que todavía funcione como un sifón?

RESPUESTA: (a) 5,0 x 103 m3/s, (b) 10 m

APUNTES DE FISICA

MsC.

JESUS ROBERTO GAVIDIA IBERICO

55

SECCION 13-6 13-40.

Durante el ejercicio pesado, la velocidad del flujo de la sangre aumenta en aproximadamente un factor de dos. Con referencia al ejemplo 13-5, calcule el número de Reynolds y determine qué tipo de flujo esperaría en la aorta.

13-41.

Calcule el número de Reynolds para el flujo de la sangre a través de un tubo capilar si su rapidez es de 4.7 x 102 cm/s.

RESPUESTA: 103 13-42.

¿Cuál es el gasto máximo (aproximado), Q, de agua en un tubo de 10 cm de diámetro si se evita que haya turbulencia?

SECCION 13-7 13-43.

Calcule la magnitud de la velocidad terminal de una burbuja de aire de 1.0 mm de radio (supuesto constante) que se eleva en un aceite de viscosidad 0.20 Pa · s y densidad especifica 0.90.

RESPUESTA: 9,8 mm/s 13-44.

¿Cuál es la velocidad terminal aproximada de una bola de acero de 2.0 cm de radio que se mueve en el aire si el flujo es laminar?

13-45.

Si un objeto necesita 30 min para sedimentarse en una ultracentrífuga que gira a 30 000 rpm a una distancia promedio del eje de 8.0 cm, ¿cuánto tardará en depositarse por efecto de la gravedad en el mismo tubo colocado verticalmente en el laboratorio?

RESPUESTA: 4,6 años 13-46.

Muestre que la velocidad terminal de una pequeña esfera de densidad 0 que cae a través de un fluido

2 (0   r ) r 2 g de densidad r y viscosidad  es: vT  (b) ¿Cuál es la velocidad terminal de una 9 gota esférica de radio r = 0.020 cm que está cayendo en el aire?

13-47.

La viscosidad, , de un fluido puede determinarse midiendo la velocidad terminal, vT, de una esfera cuando cae a través del líquido. Determine una fórmula para  en términos del radio r, la densidad , de la esfera y la densidad, r, del fluido. Suponga que no hay turbulencia.

RESPUESTA: 2 (  f)gr2/9 vT 13-48.

(a) Muestre que la fuerza de empuje sobre un pequeño objeto en el líquido de una centrífuga que gira con velocidad angular  está dado por: FB   r V  2 r donde V es el volumen del objeto, r, la distancia a su eje de rotación y r la densidad del liquido, (b) compare la dependencia de FB con la de la fuerza de empuje sobre un objeto que se sedimenta por la acción de la gravedad.

13-49.

Demuestre que la presión de un fluido en una centrífuga que gira, una distancia r, desde el eje de rotación, se expresa mediante: P 

1   2 (r 2  r02 ) en la que r es la densidad del fluido, r0 es la 2 r

distancia desde el eje de rotación de la superficie del fluido en el tubo y  es la velocidad angular.